Unit 3 Quizzes

Pataasin ang iyong marka sa homework at exams ngayon gamit ang Quizwiz!

8. Which patient is most at risk for developing delirium? a. A 50-yr-old woman with cholecystitis b. A 19-yr-old man with a fractured femur c. A 42-yr-old woman having an elective hysterectomy d. A 78-yr-old man admitted to the medical unit with complications related to heart failure

8. Correct answer: d Rationale: Risk factors that can precipitate delirium include age of 65 years or older, male gender, and severe acute illness (e.g., heart failure). The 78-year-old man has the most risk factors for delirium (Table 59-16).

What would the nurse tell the parents is the acceptable range of heart beats per minute for a preschooler? 60-100 80-110 75-100 90-140

80-110 The acceptable range for a heart rate in preschoolers is 80 to 110 beats per minute. Adults have a range of 60 to 100 beats per minute. In school-aged children, the heart rate is from 75 to 100 beats per minute. The acceptable range of heart beats per minute in a toddler is 90 to 140 beats per minute.

You are caring for a patient receiving D5W at a rate of 125 mL/hr. During the 4:00 PM assessment of the patient, you determine that 500 mL is left in the present IV bag. At what time should the nurse anticipate hanging the next bag of D5W?__

8:00 PM Divide the 500 mL left in the IV bag by the hourly rate of 125 mL to calculate that the present solution will remain infusing for another 4 hours. If you made this notation at 4:00 PM, the bag is due to be changed at 8:00 PM.

1. To promote the release of surfactant, the nurse encourages the patient to a. take deep breaths. b. cough five times per hour to prevent alveolar collapse. c. decrease fluid intake to reduce fluid accumulation in the alveoli. d. sit with head of bed elevated to promote air movement through the pores of Kohn.

1. Correct answer: a Rationale: Surfactant is a lipoprotein that lowers the surface tension in the alveoli. It reduces the amount of pressure needed to inflate the alveoli and decreases the tendency of the alveoli to collapse. Deep breaths stretch the alveoli and promote surfactant secretion.

1. A patient with a tricuspid valve disorder will have impaired blood flow between the a. vena cava and right atrium. b. left atrium and left ventricle. c. right atrium and right ventricle. d. right ventricle and pulmonary artery.

1. Correct answer: c Rationale: The tricuspid valve is located between the right atrium and right ventricle.

1. Dementia is defined as a a. syndrome that results only in memory loss. b. disease associated with abrupt changes in behavior. c. disease that is always due to reduced blood flow to the brain. d. syndrome characterized by cognitive dysfunction and loss of memory.

1. Correct answer: d Rationale: Dementia is a syndrome characterized by dysfunction in or loss of memory, orientation, attention, language, judgment, and reasoning. Personality changes and behavioral problems such as agitation, delusions, and hallucinations may result.

10. Which assessment finding of the respiratory system does the nurse interpret as abnormal? a. Inspiratory chest expansion of 1 inch b. Symmetric chest expansion and contraction c. Resonance (to percussion) over the lung bases d. Bronchial breath sounds in the lower lung fields

10. Correct answer: d Rationale: Vesicular breath sounds are most commonly auscultated over the peripheral lung fields. Bronchial or bronchovesicular sounds heard in the peripheral lung fields would be abnormal.

When planning the care of a patient with dehydration, what urine output would the nurse instruct the unlicensed assistive personnel to report? 60 mL in 90 minutes 1200 mL in 24 hours 300 mL per 8-hour shift 20 mL for 2 consecutive hours

20 mL for 2 consecutive hours The minimal urine output necessary to maintain kidney function is 30 mL/hr. If the output is less than this for 2 consecutive hours, the nurse should be notified so that additional fluid volume replacement therapy can be instituted.

When planning care for stable adult patients, the oral intake that is adequate to meet daily fluid needs is 500 to 1500 mL. 1200 to 2200 mL. 2000 to 3000 mL. 3000 to 4000 mL.

2000 to 3000 mL. Daily fluid intake and output is usually 2000 to 3000 mL. This is sufficient to meet the needs of the body and replace both sensible and insensible fluid losses. These would include urine output and fluids lost through the respiratory system, skin, and GI tract.

A patient was admitted for a paracentesis to remove ascites fluid. Five liters of fluid was removed. Which IV solution may be used to pull fluid into the intravascular space after the paracentesis? 0.9% sodium chloride 25% albumin solution Lactated Ringer's solution 5% dextrose in 0.45% saline

25% albumin solution After a paracentesis of 5 L or greater of ascites fluid, 25% albumin solution may be used as a volume expander. Normal saline, lactated Ringer's solution, and 5% dextrose in 0.45% saline will not be effective for this action.

3. The portion of the vascular system responsible for hemostasis is the a. thin capillary vessels. b. endothelial layer of the arteries. c. elastic middle layer of the veins. d. smooth muscle of the arterial wall.

3. Correct answer: b Rationale: The innermost lining of the arteries is the endothelium. The endothelium maintains hemostasis, promotes blood flow, and under normal conditions, inhibits blood coagulation.

3. The nurse can best determine adequate arterial oxygenation of the blood by assessing a. heart rate. b. hemoglobin level. c. arterial oxygen partial pressure. d. arterial carbon dioxide partial pressure.

3. Correct answer: c Rationale: The ability of the lungs to oxygenate arterial blood adequately is determined by examination of the partial pressure of oxygen in arterial blood (PaO2) and arterial oxygen saturation (SaO2).

9. When auscultating the chest of an older patient in respiratory distress, it is best to a. begin listening at the apices. b. begin listening at the lung bases. c. begin listening on the anterior chest. d. Ask the patient to breathe through the nose with the mouth closed.

9. Correct answer: b Rationale: Normally, auscultation should proceed from the lung apices to the bases so that opposite areas of the chest are compared. If the patient is likely to tire easily or has respiratory distress, start at the bases.

9. When assessing the cardiovascular system of a 79-year-old patient, you may expect to find a. a narrowed pulse pressure. b. diminished carotid artery pulses. c. difficulty in isolating the apical pulse. d. an increased heart rate in response to stress.

9. Correct answer: c Rationale: Myocardial hypertrophy and the downward displacement of the heart in an older adult may cause difficulty in isolating the apical pulse.

Which patient is at greatest risk for developing hypermagnesemia? a. 83-year-old man with lung cancer and hypertension b. 65-year-old woman with hypertension taking β-adrenergic blockers c. 42-year-old woman with systemic lupus erythematosus and renal failure d. 50-year-old man with benign prostatic hyperplasia and a urinary tract infection

c. 42-year-old woman with systemic lupus erythematosus and renal failure Rationale: Causes of hypermagnesemia include renal failure (especially if the patient is given magnesium products), excessive administration of magnesium for treatment of eclampsia, and adrenal insufficiency.

The nurse should be alert for which manifestations in a patient receiving a loop diuretic? a. Restlessness and agitation b. Paresthesias and irritability c. Weak, irregular pulse and poor muscle tone d. Increased blood pressure and muscle spasms

c. Weak, irregular pulse and poor muscle tone Rationale: Loop diuretics may result in renal loss of potassium and hypokalemia. Clinical manifestations of hypokalemia include fatigue, muscle weakness, leg cramps, nausea, vomiting, paralytic ileus, paresthesias, decreased reflexes, weak, irregular pulse, polyuria, hyperglycemia, and ECG changes.

The typical fluid replacement for the patient with a fluid volume deficit is a. dextran. b. 0.45% saline. c. lactated Ringer's. d. 5% dextrose in 0.45% saline.

c. lactated Ringer's. Rationale: Administration of an isotonic solution expands only the extracellular fluid (ECF). There is no net loss or gain from the intracellular fluid (ICF). An isotonic solution is the ideal fluid replacement for a patient with an ECF volume deficit. Examples of isotonic solutions include lactated Ringer's solution and 0.9% NaCl.

During the assessment in the emergency department, the nurse is palpating the patient's chest. Which finding is a medical emergency? Increased tactile fremitus Diminished chest movement Tracheal deviation to the left Decreased anteroposterior (AP) diameter

Tracheal deviation to the left Tracheal deviation is a medical emergency when it is caused by a tension pneumothorax. Tactile fremitus increases with pneumonia or pulmonary edema and decreases in pleural effusion or lung hyperinflation. Diminished chest movement occurs with barrel chest, restrictive disease, and neuromuscular disease.

It is important for the nurse to assess for which clinical manifestation(s) in a patient who has just undergone a total thyroidectomy (select all that apply)? a. Confusion b. Weight gain c. Depressed reflexes d. Circumoral numbness e. Positive Chvostek's sign

a. Confusion d. Circumoral numbness e. Positive Chvostek's sign Rationale: Inadvertent removal of a portion of or injury to the parathyroid glands during thyroid or neck surgery can result in a lack of parathyroid hormone, leading to hypocalcemia. A positive Chvostek's sign, confusion, and circumoral numbness are manifestations of low serum calcium levels.

A 67-yr-old male patient had a right total knee replacement 2 days ago. Upon auscultation of the patient's posterior chest, the nurse detects discontinuous, high-pitched breath sounds just before the end of inspiration in the lower portion of both lungs. Which statement most appropriately reflects how the nurse should document the breath sounds? "Bibasilar wheezes present on inspiration." "Diminished breath sounds in the bases of both lungs." "Fine crackles posterior right and left lower lung fields." "Expiratory wheezing scattered throughout the lung fields."

"Fine crackles posterior right and left lower lung fields." Fine crackles are described as a series of short-duration, discontinuous, high-pitched sounds heard just before the end of inspiration.

A patient has acute gastroenteritis with watery diarrhea. Which statement by this patient would indicate that the nurse's teaching has been effective? "I should drink a lot of tap water today." "I need to take more calcium tablets today." "I should avoid fruits with potassium in them." "I need to drink liquids with some sodium in them."

"I need to drink liquids with some sodium in them."

The patient informs the nurse that he does not understand how there can be a blockage in the left anterior descending artery (LAD), but there is damage to the right ventricle. What is the best response by the nurse? "The one vessel curves around from the left side to the right ventricle." "The LAD supplies blood to the left side of the heart and part of the right ventricle." "The right ventricle is supplied during systole primarily by the right coronary artery." "It is actually on your right side of the heart, but we call it the left anterior descending vessel."

"The LAD supplies blood to the left side of the heart and part of the right ventricle." The best response is explaining that the lower portion of the right ventricle receives blood flow from the left anterior descending artery as well as the right coronary artery during diastole.

Which instruction by the nurse is given to a patient who is about to undergo Holter monitoring is most appropriate? "You may remove the monitor only to shower or bathe." "You should connect the monitor whenever you feel symptoms." "You should refrain from exercising while wearing this monitor." "You will need to keep a diary of all your activities and symptoms."

"You will need to keep a diary of all your activities and symptoms." A Holter monitor is worn continuously for at least 24 hours while a patient continues with usual activity and keeps a diary of activities and symptoms. The patient should not take a bath or shower while wearing this monitor.

10. Which nursing responsibilities are priorities when caring for a patient returning from a cardiac catheterization (select all that apply)? a. Monitoring vital signs and ECG b. Checking the catheter insertion site and distal pulses c. Assisting the patient to ambulate to the bathroom to void d. Informing the patient that he will be sleepy from the general anesthesia e. Instructing the patient about the risks of the radioactive isotope injection

10. Correct answers: a, b Rationale: The nursing responsibilities after cardiac catheterization include assessment of the puncture site for hematoma and bleeding; assessment of circulation to the extremity used for catheter insertion and of peripheral pulses, color, and sensation of the extremity; and monitoring vital signs and electrocardiographic rhythm.

The nurse is interpreting a tuberculin skin test (TST) for a 58-yr-old female patient with end-stage renal disease secondary to diabetes mellitus. Which finding would indicate a positive reaction? Acid-fast bacilli cultured at the injection site 15-mm area of redness at the TST injection site 11-mm area of induration at the TST injection site Wheal formed immediately after intradermal injection

11-mm area of induration at the TST injection site An area of induration 10 mm or larger would be a positive reaction in a person with end-stage renal disease. Reddened, flat areas do not indicate a positive reaction. A wheal appears when the TST is administered that indicates correct administration of the intradermal antigen. Presence of acid-fast bacilli in the sputum indicates active tuberculosis.

11. The nurse is preparing the patient for a diagnostic procedure to remove pleural fluid for analysis. The nurse would prepare the patient for which test? a. Thoracentesis b. Bronchoscopy c. Pulmonary angiography d. Sputum culture and sensitivity

11. Correct answer: a Rationale: Thoracentesis is the insertion of a large-bore needle through the chest wall into the pleural space to obtain specimens for diagnostic evaluation, remove pleural fluid, or instill medication.

2. Vascular dementia is associated with a. transient ischemic attacks. b. bacterial or viral infection of neuronal tissue. c. cognitive changes secondary to cerebral ischemia. d. abrupt changes in cognitive function that are irreversible.

2. Correct answer: c Rationale: Vascular dementia is the loss of cognitive function that results from ischemic, ischemic-hypoxic, or hemorrhagic brain lesions caused by cardiovascular disease. In this type of dementia, narrowing and blocking of arteries that supply the brain cause a decrease in blood supply.

2. A patient with a respiratory condition asks, "How does air get into my lungs?" The nurse bases her answer on knowledge that air moves into the lungs because of a. increased CO2 and decreased O2 in the blood. b. contraction of the accessory abdominal muscles. c. stimulation of the respiratory muscles by the chemoreceptors. d. decrease in intrathoracic pressure relative to pressure at the airway.

2. Correct answer: d Rationale: During inspiration, the diaphragm contracts, moves downward, and increases intrathoracic volume. At the same time, the external intercostal muscles and scalene muscles contract, increasing the lateral and anteroposterior dimension of the chest. This causes the size of the thoracic cavity to increase and intrathoracic pressure to decrease, so air is pulled into the lungs.

A patient has a severe blockage in his right coronary artery. Which heart structures are most likely to be affected by this blockage (select all that apply)? a. AV node b. Left ventricle c. Coronary sinus d. Right ventricle e. Pulmonic valve

2. Correct answers: a, b, d Rationale: The right coronary artery (RCA) supplies blood to the right atrium, the right ventricle, and a portion of the posterior wall of the left ventricle. In 90% of people, the RCA supplies blood to the atrioventricular (AV) node, bundle of His, and part of the cardiac conduction system.

3. The clinical diagnosis of dementia is based on a. CT or MRS. b. brain biopsy. c. electroencephalogram. d. patient history and cognitive assessment.

3. Correct answer: d Rationale: The diagnosis of dementia depends on determining the cause. A thorough physical examination is performed to rule out other potential medical conditions. Cognitive testing (e.g., Mini-Mental State Examination) is focused on evaluating memory, ability to calculate, language, visual-spatial skills, and degree of alertness. Diagnosis of dementia related to vascular causes is based on the presence of cognitive loss, the presence of vascular brain lesions demonstrated by neuroimaging techniques, and the exclusion of other causes of dementia. Structural neuroimaging with CT scan or MRI is used in the evaluation of patients with dementia but does not provide a definitive diagnosis. A psychologic evaluation is also indicated to determine the presence of depression.

Which serum potassium result best supports the rationale for administering a stat dose of IV potassium chloride 20 mEq in 200 mL of normal saline over 2 hours? 3.1 mEq/L 3.9 mEq/L 4.6 mEq/L 5.3 mEq/L

3.1 mEq/L The normal range for serum potassium is 3.5 to 5.0 mEq/L. This IV order provides a substantial amount of potassium. Thus the patient's potassium level must be low. The only low value shown is 3.1 mEq/L.

4. When a person's blood pressure rises, the homeostatic mechanism to compensate for an elevation involves stimulation of a. baroreceptors that inhibit the sympathetic nervous system, causing vasodilation. b. chemoreceptors that inhibit the sympathetic nervous system, causing vasodilation. c. baroreceptors that inhibit the parasympathetic nervous system, causing vasodilation. d. chemoreceptors that stimulate the sympathetic nervous system, causing an increased heart rate.

4. Correct answer: a Rationale: Baroreceptors in the aortic arch and carotid sinus are sensitive to stretch or pressure within the arterial system. Stimulation of these receptors sends information to the vasomotor center in the brainstem. This results in temporary inhibition of the sympathetic nervous system and enhancement of the parasympathetic influence, which cause a decrease in heart rate and peripheral vasodilation.

4. When teaching a patient about the most important respiratory defense mechanism distal to the respiratory bronchioles, which topic would the nurse discuss? a. Alveolar macrophages b. Impaction of particles c. Reflex bronchoconstriction d. Mucociliary clearance mechanism

4. Correct answer: a Rationale: Respiratory defense mechanisms are efficient in protecting the lungs from inhaled particles, microorganisms, and toxic gases. Because ciliated cells are not found below the level of the respiratory bronchioles, the primary defense mechanism at the alveolar level is alveolar macrophages.

4. Dementia with Lewy bodies (DLB) is characterized by a. remissions and exacerbations over many years. b. memory impairment, muscle jerks, and blindness. c. parkinsonian symptoms, including muscle rigidity. d. increased intracranial pressure secondary to decreased CSF drainage.

4. Correct answer: c Rationale: Dementia with Lewy bodies (DLB) is characterized by features of dementia and Parkinson's disease. These patients typically have manifestations of parkinsonism, hallucinations, short-term memory loss, unpredictable cognitive shifts, and sleep disturbances.

The nurse is caring for a patient with chronic obstructive pulmonary disorder (COPD) and pneumonia who has an order for arterial blood gases to be drawn. What is the minimum length of time the nurse should plan to hold pressure on the puncture site? 2 minutes 5 minutes 10 minutes 15 minutes

5 minutes After obtaining blood for an arterial blood gas measurement, the nurse should hold pressure on the puncture site for 5 minutes by the clock to be sure that bleeding has stopped. An artery is an elastic vessel under much higher pressure than veins, and significant blood loss or hematoma formation could occur if the time is insufficient.

5. Which statement(s) accurately describe(s) mild cognitive impairment (select all that apply)? a. Always progresses to AD b. Caused by variety of factors and may progress to AD c. Should be aggressively treated with acetylcholinesterase drugs d. Caused by vascular infarcts that, if treated, will delay progression to AD e. Patient is usually not aware that there is a problem with his or her memory

5. Correct answer: b Rationale: Although some individuals with mild cognitive impairment (MCI) revert to normal cognitive function or do not go on to develop Alzheimer's disease (AD), those with MCI are at high risk for AD. No drugs have been approved for the treatment of MCI. A person with MCI is often aware of a significant change in memory.

5. A P wave on an ECG represents an impulse arising at the a. SA node and repolarizing the atria. b. SA node and depolarizing the atria. c. AV node and depolarizing the atria. d. AV node and spreading to the bundle of His.

5. Correct answer: b Rationale: The first wave, P, begins with the firing of the sinoatrial (SA) node and represents depolarization of the fibers of the atria.

5. A student nurse asks the RN what can be measured by arterial blood gas (ABG). The RN tells the student that the ABG can measure (select all that apply) a. acid-base balance. b. oxygenation status. c. acidity of the blood. d. bicarbonate (HCO3−) in arterial blood. e. overall balance of electrolytes in arterial blood.

5. Correct answers: a, b, c, d Rationale: Arterial blood gases (ABGs) are measured to determine oxygenation status, ventilation status, and acid-base balance. ABG analysis includes measurement of the partial pressure of oxygen in arterial blood (PaO2), partial pressure of carbon dioxide in arterial blood (PaCO2), acidity (pH), bicarbonate (HCO3-), and arterial oxygen saturation (SaO2) in arterial blood. The overall balance of electrolytes cannot be determined with ABGs.

6. The early stage of AD is characterized by a. no noticeable change in behavior. b. memory problems and mild confusion. c. increased time spent sleeping or in bed. d. incontinence, agitation, and wandering behavior.

6. Correct answer: b Rationale: An initial sign of AD is a subtle deterioration in memory.

6. To detect early signs or symptoms of inadequate oxygenation, the nurse would examine the patient for a. dyspnea and hypotension. b. apprehension and restlessness. c. cyanosis and cool, clammy skin. d. increased urine output and diaphoresis.

6. Correct answer: b Rationale: Early symptoms of inadequate oxygenation include unexplained restlessness, apprehension, and irritability.

6. When collecting subjective data related to the cardiovascular system, which information should be obtained from the patient (select all that apply)? a. Annual income b. Smoking history c. Religious preference d. Number of pillows used to sleep e. Blood for basic laboratory studies

6. Correct answers: b, c, d Rationale: The health history should include assessment of tobacco use. The patient should be asked about any cultural or religious beliefs that may influence the management of the cardiovascular problem. Patients with heart failure may need to sleep with the head elevated on pillows or sleep in a chair.

The patient's blood pressure is 90/50 mm Hg. The nurse calculates the mean arterial pressure (MAP) to see if the blood pressure is high enough to adequately perfuse and sustain the vital organs. What is the MAP?__

63 The MAP is 63. 90 + 2(50)/3 = 63. So the blood pressure is high enough to perfuse and sustain vital organs because an MAP greater than 60 is needed.

7. A priority goal of treatment for the patient with AD is to a. maintain patient safety. b. maintain or increase body weight. c. return to a higher level of self-care. d. enhance functional ability over time.

7. Correct answer: a Rationale: The overall management goals are that the patient with AD will (1) maintain functional ability for as long as possible, (2) be maintained in a safe environment with a minimum of injuries, (3) have personal care needs met, and (4) have dignity maintained. The nurse should place emphasis on patient safety while planning and providing nursing care.

7. The auscultatory area in the left midclavicular line at the level of the fifth ICS is the best location to hear sounds from which heart valve? a. Aortic b. Mitral c. Tricuspid d. Pulmonic

7. Correct answer: b Rationale: The mitral valve can be assessed by auscultation at the left midclavicular line at the fifth intercostal space (ICS).

7. During the respiratory assessment of an older adult, the nurse would expect to find (select all that apply) a. a vigorous reflex cough. b. increased chest expansion. c. increased residual volume. d. diminished lung sounds at base of lungs. e. increased anteroposterior (AP) chest diameter.

7. Correct answers: c, d, e Rationale: The anterior-posterior diameter of the thoracic cage and the residual volume increase in older adults. An older adult has a less forceful cough. The costal cartilages calcify with aging and interfere with chest expansion. Decreased breath sounds at the base of lungs is also a common finding in older adults.

8. When assessing a patient, you note a pulse deficit of 23 beats. This finding may be caused by a. dysrhythmias. b. heart murmurs. c. gallop rhythms. d. pericardial friction rubs.

8. Correct answer: a Rationale: A pulse deficit occurs if there is a difference between the apical and radial beats per minute. A pulse deficit indicates cardiac dysrhythmias.

8. When assessing activity-exercise patterns related to respiratory health, the nurse inquires about a. dyspnea during rest or exercise. b. recent weight loss or weight gain. c. ability to sleep through the entire night. d. willingness to wear O2 equipment in public.

8. Correct answer: a Rationale: In this functional health pattern, determine whether the patient's activity is limited by dyspnea at rest or during exercise.

Which patient is exhibiting an early clinical manifestation of hypoxemia? A 48-yr-old patient who is intoxicated and acutely disoriented to time and place A 67-yr-old patient who has dyspnea while resting in the bed or in a reclining chair A 72-yr-old patient who has four new premature ventricular contractions per minute A 94-yr-old patient who has renal insufficiency, anemia, and decreased urine output

A 72-yr-old patient who has four new premature ventricular contractions per minute Early clinical manifestations of hypoxemia include dysrhythmias (e.g., premature ventricular contractions), unexplained decreased level of consciousness (e.g., disorientation), dyspnea on exertion, and unexplained decreased urine output.

Which diagnosis indicates that the nurse should assess the patient most carefully for development of metabolic acidosis? Type B chronic obstructive pulmonary disease (COPD) and pneumonia Acute meningococcal meningitis A pancreatic fistula that is draining Severe hyperaldosteronism

A pancreatic fistula that is draining The pancreas secretes bicarbonate; a draining pancreatic fistula could cause metabolic acidosis from bicarbonate loss. Type B COPD and pneumonia cause respiratory acidosis by impairing carbonic acid excretion. Meningitis can stimulate hyperventilation, which causes respiratory alkalosis. Aldosterone facilitates renal excretion of hydrogen ions; hyperaldosteronism would cause metabolic alkalosis.

The nurse would anticipate that which of the following patients will need to be treated with insertion of a chest tube? A patient with asthma and severe shortness of breath A patient undergoing a bronchoscopy for a biopsy A patient with a pleural effusion requiring fluid removal A patient experiencing a problem with a pneumothorax

A patient experiencing a problem with a pneumothorax When air is allowed to enter the pleural space, the lung will collapse and a chest tube will be inserted to remove the air and reestablish negative pressure in the pleural space. Patients with asthma do not require a chest tube. A bronchoscopy is done to evaluate the bronchi and lungs and to obtain a biopsy. A thoracentesis may be done to remove fluid from the pleural space. A chest tube may be inserted if there are complications from the thoracentesis or for the bronchoscopy.

The nurse on a medical-surgical unit identifies which patient as having the highest risk for metabolic alkalosis? A patient with a traumatic brain injury A patient with type 1 diabetes mellitus A patient with acute respiratory failure A patient with nasogastric tube suction

A patient with nasogastric tube suction Excessive nasogastric suctioning may cause metabolic alkalosis. Brain injury may cause hyperventilation and respiratory alkalosis. Type 1 diabetes mellitus (diabetic ketoacidosis) is associated with metabolic acidosis. Acute respiratory failure may lead to respiratory acidosis.

The nurse knows that including teaching on modifiable risk factors for impaired perfusion in the patient's plan of care includes which of the following: Impaired perfusion increases with age. Genetics play a role in impaired perfusion. Exercise should be kept at a minimum to prevent a myocardial infarction. A smoking cessation plan should be in place.

A smoking cessation plan should be in place. The importance of distinguishing between modifiable versus nonmodifiable risk factors is imperative when determining what sort of lifestyle changes can be discussed when formulating the patient's plan of care. Impaired perfusion can affect all people and age groups regardless of gender, race, or economic status. Smoking cessation is an example of a modifiable risk factor for impaired perfusion that can be included in the patient's plan of care. Modifiable risk factors can be changed by the patient through teaching from the nurse. Although impaired perfusion can increase with age, this is an example of an unmodifiable risk factor (something that the patient cannot change). Genetics is an example of an unmodifiable risk factor for impaired perfusion. A sedentary lifestyle can lead to obesity, which would then become a modifiable risk factor for impaired perfusion.

When assessing the patient with a multi-lumen central line, the nurse notices that the cap is off one of the lines. On assessment, the patient is in respiratory distress and the vital signs show hypotension and tachycardia. What is the nurse's priority action? Administer oxygen Notify the health care provider Rapidly administer more IV fluid Reposition the patient on the right side

Administer oxygen The cap off the central line could allow entry of air into the circulation, causing an air embolus. To manage an air embolus, oxygen is administered; the catheter is clamped, and the patient is positioned on the left side with the head down. Then the health care provider is notified.

According to available research, which is a primary risk factor for cognitive impairment? Advancing age Female gender Caucasian Northern European ancestry

Advancing age Multiple studies have been conducted evaluating characteristics of individuals who developed cognitive impairment compared with those who did not. The results indicate that a primary risk factor for cognitive impairment is advancing age. No differences in impairment have been found across populations based on race, ethnicity, or gender, although correlated risk factors among women and men differ.

A client has a pulse deficit. Which documentation by the nurse supports this finding? Blood pressure of 130/70 mm Hg indicating pulse deficit of 60. Capillary refill greater than 3 seconds indicating pulse deficit. Apical pulse 86 and radial pulse 78 indicating pulse deficit of 8. Radial pulse 80 and pedal pulse 70 indicating pulse deficit of 10.

Apical pulse 86 and radial pulse 78 indicating pulse deficit of 8. The apical rate is more rapid than the radial rate when a pulse deficit exists. An apical pulse of 86 with a radial pulse of 78 is a pulse deficit of 8. A blood pressure of 130/70 mm Hg is a pulse pressure of 60. Capillary refill greater than 3 seconds indicates circulation is sluggish. Radial pulse of 80 and a pedal pulse of 70 do not indicate a pulse deficit; a pulse deficit is the difference between the apical and peripheral pulses.

Which of the following tools is the best measure to determine a patient's gas exchange and respitary function? Chest x-ray Oxygen saturation Arterial blood gas (ABG) analysis Central venous pressure monitoring

Arterial blood gas (ABG) analysis ABG analysis is most useful in this setting because ventilatory failure causes problems with CO2 retention, and ABGs provide information about the PaCO2 and pH. The other tests may also be done to help in assessing oxygenation or determining a patient's ventilatory function; however, they are not the best measure.

The patient is hospitalized with pneumonia. Which diagnostic test should be used to measure the efficiency of gas exchange in the lung and tissue oxygenation? Thoracentesis Bronchoscopy Arterial blood gases Pulmonary function tests

Arterial blood gases Arterial blood gases are used to assess the efficiency of gas exchange in the lung and tissue oxygenation as is pulse oximetry. Thoracentesis is used to obtain specimens for diagnostic evaluation, remove pleural fluid, or instill medication into the pleural space. Bronchoscopy is used for diagnostic purposes, to obtain biopsy specimens, and to assess changes resulting from treatment. Pulmonary function tests measure lung volumes and airflow to diagnose pulmonary disease, monitor disease progression, evaluate disability, and evaluate response to bronchodilators.

The patient with Parkinson's disease has a pulse oximetry reading of 72%, but he is not displaying any other signs of decreased oxygenation. What is most likely contributing to his low SpO2 level? Artifact Anemia Dark skin color Thick acrylic nails

Artifact Motion is the most likely cause of the low SpO2 for this patient with Parkinson's disease. Anemia, dark skin color, and thick acrylic nails as well as low perfusion, bright fluorescent lights, and intravascular dyes may also cause an inaccurate pulse oximetry result. There is no mention of these or reason to suspect these in this question.

A nurse is caring for a patient immediately following a transesophageal echocardiogram (TEE). Which assessments are appropriate for this patient (select all that apply.)? Assess for return of gag reflex. Assess groin for hematoma or bleeding. Monitor vital signs and oxygen saturation. Position patient supine with head of bed flat. Assess lower extremities for circulatory compromise.

Assess for return of gag reflex. Monitor vital signs and oxygen saturation. The patient undergoing a TEE has been given conscious sedation and has had the throat numbed with a local anesthetic spray, thus eliminating the gag reflex until the effects wear off. Therefore it is imperative that the nurse assess for gag reflex return before allowing the patient to eat or drink. Vital signs and oxygen saturation are also important assessment parameters resulting from the use of sedation. A TEE does not involve invasive procedures of the circulatory blood vessels. Therefore it is not necessary to monitor the patient's groin and lower extremities in relation to this procedure or to maintain a flat position.

The nurse is caring for a patient who suddenly becomes agitated and confused. Which action should the nurse takes first? Notify the health care provider. Check pupils for reaction to light. Attempt to calm and reorient the patient. Assess oxygenation using pulse oximetry.

Assess oxygenation using pulse oximetry. Because agitation and confusion are frequently the initial indicators of hypoxemia, the nurse's initial action should be to assess oxygen saturation. The other actions are also appropriate, but assessment of oxygenation takes priority over other assessments and notification of the health care provider.

In palpating the patient's pedal pulses, the nurse determines the pulses are absent. What factor could contribute to this result? Atherosclerosis Hyperthyroidism Arteriovenous fistula Cardiac dysrhythmias

Atherosclerosis Atherosclerosis can cause an absent peripheral pulse. The feet would also be cool and may be discolored. Hyperthyroidism causes a bounding pulse. Arteriovenous fistula gives a thrill or vibration to the vessel, although this would not be in the foot. Cardiac dysrhythmias cause an irregular pulse rhythm.

Which aspects of cognitive function are tested when a person is asked to start with 100 and count backward, subtracting 7 each time? Mark all that apply. Attention Concentration Thought process Immediate recall Short-term memory Long term memory

Attention Concentration Asking the patient to count backward from 100, subtracting 7 each time is an assessment of attention and concentration. Attention and immediate recall are tested by asking the patient to repeat a set of numbers both as stated and backward. Thought process is assessed by evaluating conversation for coherence, relevance, logic, and organization. Short-term memory is assessed by asking the patient to remember three stated items and repeat them back in 5 minutes. Long-term memory is tested by asking for information that has been in memory for at least 24 hours.

The nurse is obtaining a focused respiratory assessment of a 44-yr-old female patient who is in severe respiratory distress 2 days after abdominal surgery. What is most important for the nurse to assess? Auscultation of bilateral breath sounds Percussion of anterior and posterior chest wall Palpation of the chest bilaterally for tactile fremitus Inspection for anterior and posterior chest expansion

Auscultation of bilateral breath sounds Important assessments obtained during a focused respiratory assessment include auscultation of lung (breath) sounds. Assessment of tactile fremitus has limited value in acute respiratory distress. It is not necessary to assess for both anterior and posterior chest expansion. Percussion of the chest wall is not essential in a focused respiratory assessment.

A frail 82-yr-old female patient develops sudden shortness of breath while sitting in a chair. What location on the chest should the nurse begin auscultation of the lung fields? Bases of the posterior chest area Apices of the posterior lung fields Anterior chest area above the breasts Midaxillary on the left side of the chest

Bases of the posterior chest area Baseline data with the most information is best obtained by auscultation of the posterior chest, especially in female patients because of breast tissue interfering with the assessment or if the patient may tire easily (e.g., shortness of breath, dyspnea, weakness, fatigue). Usually auscultation proceeds from the lung apices to the bases unless it is possible the patient will tire easily. In this case, the nurse should start at the bases.

In assessing the patient for cardiovascular abnormalities, the nurse is aware that the blood flows through the heart in what order? (Answer with a letter followed by a comma and a space (e.g. A, B, C, D).) __ a. Lungs b. Left atrium c. Right atrium d. Mitral valve e. Left ventricle f. Right ventricle

C, F, A, B, D, E Blood flows (from the body via the superior and inferior vena cava) into the right atrium. Blood passes from the right atrium (through the tricuspid valve) into the right ventricle, then (via the pulmonic valve) into the pulmonic artery and the lungs. Blood flows back from the lungs (via the pulmonary veins) into the left atrium. Blood flows from the left atrium through the mitral valve into the left ventricle and then (via the aortic valve) to the aorta and the body.

The nurse hears a series of long, discontinuous low-pitched sounds similar to blowing through a straw under water while auscultating the lungs of a client with chronic obstructive pulmonary disease. What should the nurse document in the client's assessment record based on this finding? Rhonchi Wheezes Fine crackles Coarse crackles

Coarse crackles -A series of long, discontinuous low-pitched sounds similar to blowing through straw under water indicates coarse crackles. -Rhonchi are continuous rumbling, snoring, or rattling sounds that occur as a result of an obstruction of the large airways. -Wheezes are continuous high-pitched squeaking or musical sounds that indicate airway obstruction. -Fine crackles are short, discontinuous, high-pitched sounds like hair being rolled between fingers just behind the ear, heard just before the end of inspiration.

The nurse, when auscultating the lower lungs of the patient, hears these breath sounds. How should the nurse document these sounds? Stridor Vesicular Coarse crackles Bronchovesicular

Coarse crackles Coarse crackles are a series of long-duration, discontinuous, low-pitched sounds caused by air passing through an airway intermittently occluded by mucus, an unstable bronchial wall, or a fold of mucosa. Coarse crackles are evident on inspiration and at times expiration. Stridor is a continuous crowing sound of constant pitch from partial obstruction of larynx or trachea. Vesicular sounds are relatively soft, low-pitched, gentle, rustling sounds. They are heard over all lung areas except the major bronchi. Bronchovesicular sounds are normal sounds heard anteriorly over the mainstem bronchi on either side of the sternum and posteriorly between the scapulae with a medium pitch and intensity.

Which of the following is an essential defining difference between delirium and dementia? Occurrence of sundowning syndrome Presence of delusions Incoherent speech Disturbance in consciousness

Disturbance in consciousness Delirium is a disorder of disturbed consciousness and altered cognition, whereas dementia is characterized by progressive deterioration in cognitive function with little or no disturbance in consciousness or perception. Sundowning, delusions, and incoherent speech occur with both conditions.

The patient is calling the clinic with a cough. What assessment should be made first before the nurse advises the patient? Frequency, family history, hematemesis Weight loss, activity tolerance, orthopnea Cough sound, sputum production, pattern Smoking status, medications, residence location

Cough sound, sputum production, pattern The sound of the cough, sputum production and description, and the pattern of the cough's occurrence (including acute or chronic) and what its occurrence is related to are the first assessments to be made to determine the severity. Frequency of the cough will not provide a lot of information. Family history can help to determine a genetic cause of the cough. Hematemesis is vomiting blood and not as important as hemoptysis. Smoking is an important risk factor for chronic obstructive pulmonary disease, and lung cancer and may cause a cough. Medications may or may not contribute to a cough as does residence location. Weight loss, activity intolerance, and orthopnea may be related to respiratory or cardiac problems, but are not as important when dealing with a cough.

A patient informs the nurse that he is having severe chest pain. He took two nitroglycerin tablets more than 1 hour ago with no relief. On further assessment the nurse notices that the patient is diaphoretic and also complains of feeling dizzy. The nurse would expect which diagnostic test or tests to be ordered: Vitamin B12 test Incorrect Cardiac stress test Creatine kinase-myoglobin (CK-MB) and C-reactive protein (CRP) Complete blood count (CBC) and basic metabolic panel (BMP)

Creatine kinase-myoglobin (CK-MB) and C-reactive protein (CRP) Pain, diaphoresis, and dizziness are all symptoms of someone experiencing acute coronary syndrome, which likely precipitates a myocardial infarction (MI). Patients with angina pectoris often take nitroglycerin to relieve the chest pain, which accompanies impaired tissue perfusion. Because the patient's chest pain is unrelieved by the nitroglycerin, this alerts the nurse that he is experiencing unstable angina, possibly advancing to MI. Enzymes and markers such as CK, present in the myocardium, are often evaluated as a diagnostic measure for an MI because enzymes that are released from damaged cells circulate in the blood and can be detected to confirm the presence of impaired perfusion. Although vitamin B12 deficiencies can elevate homocysteine (Hcy) levels, it is more likely that Hcy levels will be measured, rather than a laboratory test for vitamin B12 levels. A cardiac stress test may be used to measure vital signs during exercise on a treadmill. A cardiac stress test is not recommended if a patient is experiencing signs of an MI. Although a CBC and BMP may be ordered to evaluate a patient's overall functioning, it would not be used as a diagnostic tool to diagnose an MI.

A 22-yr-old man is admitted to the emergency department with a stab wound to the abdomen. The patient's vital signs are blood pressure 82/56 mm Hg, pulse 132 beats/min, respirations 28 breaths/min, and temperature 97.9° F (36.6° C). Which fluid, if ordered by the health care provider, should the nurse question? D5W 0.9% saline Packed red blood cells Lactated Ringer's solution

D5W IV administration of 0.45% saline is hypotonic and is used for maintenance fluid replacement and dilutes the extracellular fluid. IV solutions used for volume expansion for hypovolemic shock include lactated Ringer's solution and 0.9% saline. If hypovolemia is due to blood loss, blood may be administered.

After swallowing, a 73-yr-old patient is coughing and has a wet voice. What changes of aging could be contributing to this abnormal finding? Decreased response to hypercapnia Decreased number of functional alveoli Increased calcification of costal cartilage Decreased respiratory defense mechanisms

Decreased respiratory defense mechanisms These manifestations are associated with aspiration, which more easily occur in the right lung as the right mainstem bronchus is shorter, wider, and straighter than the left mainstem bronchus. Aspiration occurs more easily in the older patient related to decreased respiratory defense mechanisms (e.g., decreases in immunity, ciliary function, cough force, sensation in pharynx). Changes of aging include a decreased response to hypercapnia, decreased number of functional alveoli, and increased calcification of costal cartilage, but these do not increase the risk of aspiration.

A patient has a tumor that secretes excessive antidiuretic hormone (ADH). He is confused and lethargic. His partner wants to know how a change in blood sodium can cause these symptoms. What should the nurse teach the patient's partner? Decreased sodium in the blood causes the blood volume to decrease so that not enough oxygen reaches the brain. Decreased sodium in the blood causes brain cells to swell so that they do not work as effectively. Increased sodium in the blood causes the blood volume to increase so that too much oxygen reaches the brain. Increased sodium in the blood causes brain cells to shrivel so that they do not work as effectively.

Decreased sodium in the blood causes brain cells to swell so that they do not work as effectively. The normal action of ADH is renal reabsorption of water, which dilutes the blood. Excessive ADH causes hyponatremia, which is manifested by a decreased level of consciousness because the osmotic shift of water into the brain cells impairs their function. Hyponatremia does not decrease the blood volume. Answers that include increased sodium in the blood are incorrect because ADH excess causes hyponatremia rather than hypernatremia.

A 90-year-old patient is admitted to the hospital. Shortly after admission, the family notices that the patient is exhibiting disorientation and agitation. When questioned about the behavior by the family, the nurse states that the patient is at risk for developing which common complication of hospitalization in older adults? Delirium Dementia Alzheimer's disease Sundowner syndrome

Delirium Delirium, which occurs over hours to a few days, is the most frequent complication of hospitalization in the elderly population. Dementia occurs over a period of months. Alzheimer's disease develops over months to years. Sundowner syndrome is most prominent in dementia and becomes worse in the evenings.

A patient recently admitted to the hospital has been diagnosed with delirium. The family of the patient asks the nurse to explain what delirium is. How should the nurse respond? Delirium is reversible with treatment of the underlying cause. Delirium is progressive and has no known cure. Delirium affects a specific area of cognitive functioning. Delirium indicates the onset of a cerebrovascular accident.

Delirium is reversible with treatment of the underlying cause. Delirium can be reversible with treatment of the precipitating problem and control of predisposing factors. Dementia is progressive and irreversible. Focal cognitive disorders affect a single area of cognitive functioning. Memory and orientation may be affected by a cerebrovascular accident (stroke), but delirium is not a sign of a stroke.

When looking at the electrocardiogram (ECG) of the patient, the nurse knows that the QRS complex recorded on the ECG represents which part of the heart's beat? Depolarization of the atria Repolarization of the ventricles Depolarization from atrioventricular (AV) node throughout ventricles The length of time it takes for the impulse to travel from the atria to the ventricles

Depolarization from atrioventricular (AV) node throughout ventricles The QRS recorded on the ECG represents depolarization from the AV node throughout the ventricles. The P wave represents depolarization of the atria. The T wave represents repolarization of the ventricles. The interval between the PR and QRS represents the length of time it takes for the impulse to travel from the atria to the ventricles.

A nurse administers oxygen at 2 L/min via nasal cannula to a client with chronic obstructive pulmonary disease (COPD). By administering a low concentration of oxygen to this client, the nurse is preventing which physiologic response? Decrease in red cell formation Rupture of emphysematous bullae Depression in the respiratory center Excessive drying of the respiratory mucosa

Depression in the respiratory center It is believed that clients with COPD should be given low concentrations of oxygen because a decreased oxygen blood level is the stimulus for breathing for these clients. However, the results of a recent study of clients with stable COPD indicate that the hypercarbic drive is preserved with oxygen concentrations higher than 2 L/min. More research is needed before this theory is applied clinically. Prolonged hypoxia stimulates erythrocyte production; the goal of therapy is to relieve hypoxia. The pressure, rather than the concentration, at which oxygen is administered increases the risk of rupture of emphysematous bullae. The concentration of oxygen is unrelated to its humidification. To prevent its drying effects on secretions and the mucosa, oxygen should be humidified.

When assessing a patient's sleep-rest pattern related to respiratory health, what should the nurse ask the patient (select all that apply.)? Do you awaken abruptly during the night? Do you sleep more than 8 hours per night? Do you need to sleep with the head elevated? Do you often need to urinate during the night? Do you toss and turn when trying to fall asleep?

Do you awaken abruptly during the night? Do you need to sleep with the head elevated? Do you toss and turn when trying to fall asleep? A patient with obstructive sleep apnea may have insomnia, abrupt awakenings, or both. Patients with cardiovascular disease (e.g., heart failure that may affect respiratory health) may need to sleep with the head elevated on several pillows (orthopnea). Sleeping more than 8 hours per night or needing to urinate during the night is not indicative of impaired respiratory health.

Which action is most important for the nurse to take when caring for a patient with a subclavian triple-lumen catheter? Change the injection cap after the administration of IV medications. Use a 5-mL syringe to flush the catheter between medications and after use. During removal of the catheter, have the patient perform the Valsalva maneuver. If resistance is met when flushing, use the push-pause technique to dislodge the clot.

During removal of the catheter, have the patient perform the Valsalva maneuver. The nurse should withdraw the catheter while the patient performs the Valsalva maneuver to prevent an air embolism. Injection caps should be changed at regular intervals but not routinely after medications. Flushing should be performed with at least a 10-mL syringe to avoid excess pressure on the catheter. If resistance is encountered during flushing, force should not be applied. The push-pause method is preferred for flushing catheters but not used if resistance is encountered during flushing.

The nurse knows that primary prevention strategies to prevent impaired perfusion in the patient include which of the following recommendations by the American Heart Association (AHA): Routine blood pressure monitoring Administering furosemide (Lasix) to a patient with active congestive heart failure (CHF) symptoms Eating a healthy diet and exercising most days of the week Monitoring routine serum lipids

Eating a healthy diet and exercising most days of the week Primary prevention strategies include measures that promote health and prevent disease from developing. The American Heart Association recommends eating a heart-healthy diet, exercising most days of the week, taking a low-dose aspirin, and not smoking. Routine blood pressure monitoring is considered secondary prevention, which also includes screening and early diagnosis of health issues.Although administering a diuretic such as furosemide to a patient who presents with active CHF symptoms is considered an optimal treatment of symptoms, this is not considered a primary prevention strategy. Testing for routine serum lipids is considered secondary prevention.

The home health nurse should assess a patient who has chronic diarrhea for which fluid and electrolyte imbalances? Extracellular fluid volume (ECV) excess Extracellular fluid volume (ECV) deficit Hypokalemia Hyperkalemia Hypocalcemia Hypercalcemia

Extracellular fluid volume (ECV) deficit Hypokalemia Hypocalcemia Chronic diarrhea has a high risk of causing ECV deficit, hypokalemia, and hypocalcemia because it increases the fecal output of sodium-containing fluid, potassium, and calcium. Unless the intake of these substances increases appropriately, imbalances will occur. Excesses of ECV, potassium, and calcium are not likely, because the ECV, potassium, and calcium are being removed from the body.

What type of an environment should the nurse provide for a confused client? Familiar Variable Challenging Nonstimulating

Familiar Sameness provides security and safety and reduces stress for the client. Confused clients have difficulty adapting to a constantly changing environment. A challenging environment will increase anxiety and frustration in a confused client. A nonstimulating environment will promote diminution of the client's intellect.

What should the nurse inspect when assessing a patient with shortness of breath for evidence of long-standing hypoxemia? Fingernails Chest excursion Spinal curvatures Respiratory pattern

Fingernails Clubbing, a sign of long-standing hypoxemia, is evidenced by an increase in the angle between the base of the nail and fingernail to 180 degrees or more, usually accompanied by an increase in the depth, bulk, and sponginess of the end of the finger.

When planning care for a patient with dehydration related to nausea and vomiting, the nurse would anticipate which fluid shift to occur because of the fluid volume deficit? Fluid movement from the blood vessels into the cells Fluid movement from the interstitial spaces into the cells Fluid movement from the blood vessels into interstitial spaces Fluid movement from the interstitial space into the blood vessels

Fluid movement from the interstitial space into the blood vessels In dehydration, fluid is lost first from the blood vessels. To compensate, fluid moves out of the interstitial spaces into the blood vessels to restore circulating volume in that compartment. As the interstitial spaces then become volume depleted, fluid moves out of the cells into the interstitial spaces.

The nurse is admitting an older adult with left-sided heart failure. The nursing assessment reveals adventitious lung sounds, dyspnea, and orthopnea. The nurse anticipates which of the following orders? Furosemide (Lasix) 20 mg PO now Intravenous (IV) 500 mL of 0.9% NaCl at 125 mL/h IV Dextrose 5% at 125 ml/h IV D KCl 20 mEq at 125 ml/h

Furosemide (Lasix) 20 mg PO now Lasix is a diuretic, which will assist in relieving extracellular fluid volume (ECV) excess, which is the major consideration with left-sided heart failure. The remaining options are incorrect because IV fluids may place an additional load on the failing heart.

When evaluating the concept of gas exchange, how would the nurse best describe the movement of oxygen and carbon dioxide? Oxygen and carbon dioxide are exchanged across the capillary membrane to provide oxygen to hemoglobin. Gas moves from an area of high pressure to an area of low pressure across the alveolar membrane. The level of inspired oxygen must be sufficient to displace the carbon dioxide molecules in the alveoli. Gases are exchanged between the atmosphere and the blood based on the oxygen-carrying capacity of the hemoglobin.

Gas moves from an area of high pressure to an area of low pressure across the alveolar membrane. Oxygen and carbon dioxide move across the alveolar membrane based on the partial pressure of each gas. Molecules of oxygen are not exchanged for molecules of carbon dioxide. The pressure gradient of each gas (carbon dioxide and oxygen) in the alveoli is responsible for the movement of each gas.

A patient with a history of myocardial infarction is scheduled for a transesophageal echocardiogram to visualize a suspected clot in the left atrium. What information should the nurse include when teaching the patient about this diagnostic study? IV sedation may be administered to help the patient relax. Food and fluids are restricted for 2 hours before the procedure. Ambulation is restricted for up to 6 hours before the procedure. Contrast medium is injected into the esophagus to enhance images.

IV sedation may be administered to help the patient relax. IV sedation is administered to help the patient relax and ease the insertion of the tube into the esophagus. Food and fluids are restricted for at least 6 hours before the procedure. Smoking and exercise are restricted for 3 hours before exercise or stress testing but not before TEE. Contrast medium is administered IV to evaluate the direction of blood flow if a septal defect is suspected.

A client with chronic obstructive pulmonary disease (COPD) states, "I have had steady weight loss, and I am often too tired to eat." Which nursing diagnosis would be most appropriate for this client? Fatigue related to weight loss secondary to COPD Imbalanced nutrition: less than body requirements, related to fatigue Imbalanced nutrition: less than body requirements, related to COPD Ineffective breathing pattern, related to alveolar hypoventilation

Imbalanced nutrition: less than body requirements, related to fatigue The response portion of the nursing diagnosis is Imbalanced nutrition: less than body requirements, and the etiology is fatigue associated with the disease process of COPD. Interventions should be planned to deal with the breathing problem and the fatigue associated with it while implementing actions to combat the weight loss. Weight loss related to COPD is not a NANDA-approved nursing diagnosis. Fatigue associated with the COPD disease process is the cause of the weight loss, not COPD in itself. Altered breathing pattern is also a problem, but does not specifically relate to the weight loss problem.

When the patient is experiencing metabolic acidosis secondary to type 1 diabetes mellitus, what physiologic response should the nurse expect to assess in the patient? Vomiting Increased urination Decreased heart rate Increased respiratory rate

Increased respiratory rate When a patient with type 1 diabetes has hyperglycemia and ketonemia causing metabolic acidosis, the physiologic response is to increase the respiratory rate and tidal volume to blow off the excess CO2. Vomiting and increased urination may occur with hyperglycemia, but not as physiologic responses to metabolic acidosis. The heart rate will increase.

The nurse is caring for a patient with chronic obstructive pulmonary disease (COPD). The patient tells the nurse he is having a "hard time breathing." His respiratory rate is 32 breaths per minute, his pulse is 120 beats per minute, and the oxygen saturation is 90%. What would be the best nursing intervention for this patient? Begin oxygen via a face mask at 60% FiO2 (fraction of inspired oxygen) Administer a PRN (as necessary) dose of an intranasal glucocorticoid Encourage coughing and deep breathing to clear the airway Initiate oxygen via a nasal cannula, and begin at a flow rate of 3 L/min

Initiate oxygen via a nasal cannula, and begin at a flow rate of 3 L/min The normal respiratory drive is a person's level of carbon dioxide (CO2) in the arterial blood. The COPD patient had compensated for his chronic high levels of CO2, and his respiratory drive is dependent on his oxygen levels, not his CO2 levels. If the COPD patient's oxygen level is rapidly increased to what would be considered a normal level, it would compensate for his respiratory drive. The patient with COPD who has difficulty breathing should be given low levels of oxygen and closely observed for the quality and rate of ventilation. A dose of glucocorticoids will not address his immediate needs, but it may provide decreased inflammation and better ventilation over an extended period of time. Encouraging coughing and deep breathing in a patient with COPD does not meet his needs as effectively as administration of low-level oxygen does.

The nurse suspects that a patient has a decreased cellular volume with a possible electrolyte imbalance. The provider has ordered blood chemistry laboratory tests. What is the most important nursing intervention for this patient until laboratory results confirm this suspicion? Raise bedside rails because of potential decreased level of consciousness and confusion. Examine sacral area and patient's heels for skin breakdown caused by potential edema. Establish seizure precautions because of potential muscle twitching, cramps, and seizures. Institute fall precautions because of potential postural hypotension and weak leg muscles.

Institute fall precautions because of potential postural hypotension and weak leg muscles. Electrolyte imbalances are abnormal plasma concentrations of electrolytes such as K+, Ca++, and Mg++. Hypokalemia can cause postural hypotension and bilateral muscle weakness, especially in the lower extremities. Both of these increase the risk of falls. Options A, B, and C are incorrect because decreased cellular volume does not cause edema, decreased level of consciousness, or seizures.

The nurse is admitting a patient who is scheduled to undergo a cardiac catheterization. What allergy information is most important for the nurse to assess and document before this procedure? Iron Iodine Aspirin Penicillin

Iodine The physician will usually use an iodine-based contrast to perform this procedure. Therefore it is imperative to know whether or not the patient is allergic to iodine or shellfish. Knowledge of allergies to iron, aspirin, or penicillin will be secondary.

The nurse is providing care for a patient who has decreased cardiac output related to heart failure. What should the nurse recognize about cardiac output? It is calculated by multiplying the patient's stroke volume by the heart rate. It is the average amount of blood ejected during one complete cardiac cycle. It is determined by measuring the electrical activity of the heart and the patient's heart rate. It is the patient's average resting heart rate multiplied by the patient's mean arterial blood pressure.

It is calculated by multiplying the patient's stroke volume by the heart rate. Cardiac output is determined by multiplying the patient's stroke volume by heart rate, thus identifying how much blood is pumped by the heart over a 1-minute period. Electrical activity of the heart and blood pressure are not direct components of cardiac output.

A patient has newly diagnosed hyperparathyroidism. What should the nurse expect to find during an assessment at the beginning of the nursing shift? Lethargy and constipation from hypercalcemia Positive Trousseau's sign from hypercalcemia Lethargy and constipation from hypocalcemia Positive Trousseau's sign from hypocalcemia

Lethargy and constipation from hypercalcemia Parathyroid hormone (PTH) shifts calcium from the bones into the extracellular fluid (ECF). Excessive PTH causes hypercalcemia, which is manifested by lethargy and constipation. A positive Trousseau's sign is characteristic of hypocalcemia rather than hypercalcemia. Answers that indicate hypocalcemia are not correct, because PTH moves calcium into the ECF.

You are caring for an older patient who is receiving IV fluids postoperatively. During the 8:00 AM assessment of this patient, you note that the IV solution, which was ordered to infuse at 125 mL/hr, has infused 950 mL since it was hung at 4:00 AM. What is the priority nursing intervention? Slow the rate to keep vein open until next bag is due at noon. Notify the health care provider and complete an incident report. Listen to the patient's lung sounds and assess respiratory status. Asses the patient's cardiovascular status by checking pulse and blood pressure.

Listen to the patient's lung sounds and assess respiratory status. After 4 hours of infusion time, 500 mL of IV solution should have infused, not 950 mL. This patient is at risk for fluid volume excess, and you should assess the patient's respiratory status and lung sounds as the priority action and then notify the health care provider for further orders.

You are caring for a patient admitted with heart failure. The morning laboratory results reveal a serum potassium level of 2.9 mEq/L. Which classification of medications should you withhold until consulting with the health care provider? Antibiotics Loop diuretics Bronchodilators Antihypertensives

Loop diuretics Loop diuretics are contraindicated during episodes of hypokalemia because these medications cause the kidneys to excrete sodium and potassium. Thus administration of this type of medication at this time would worsen the hypokalemia, putting the patient at risk for dysrhythmias. The prescribing physician should be consulted for potassium replacement therapy, and the drug should be withheld until the potassium has returned to normal range.

The blood pressure of an older adult patient admitted with pneumonia is 160/70 mm Hg. What is an age-related change that contributes to this finding? Stenosis of the heart valves Decreased adrenergic sensitivity Increased parasympathetic activity Loss of elasticity in arterial vessels

Loss of elasticity in arterial vessels An age-related change that increases the risk of systolic hypertension is a loss of elasticity in the arterial walls. Because of the increasing resistance to flow, pressure is increased within the blood vessel, and hypertension results. Valvular rigidity of aging causes murmurs, and decreased adrenergic sensitivity slows the heart rate. Blood pressure is not raised. Increased parasympathetic activity would slow the heart rate.

The nurse is assessing a patient who has diabetic ketoacidosis. Her assessment reveals tachycardia, lethargy, and hyperventilation. Treatment for the ketoacidosis has been initiated. What should the nurse do about the hyperventilation? Request an order for pain medication and oxygen at 6 L/min. Lubricate the patient's lips and allow continued hyperventilation. Have the patient breathe into a paper bag to stop hyperventilating. Contact the physician immediately regarding this complication.

Lubricate the patient's lips and allow continued hyperventilation. Hyperventilation is a compensatory response to metabolic acidosis and should be allowed to continue because it helps move the blood pH toward the normal range. Lubricating the lips is a supportive nursing intervention that prevents drying and cracking of the lips during hyperventilation. Although pain and hypoxia can trigger hyperventilation, they are not the cause in this patient. Interventions to stop hyperventilation are not appropriate when it is a compensatory response. Hyperventilation is an expected beneficial compensatory response to metabolic acidosis and does not require contacting the physician.

A dehydrated patient is receiving a hypertonic solution. Which assessments must be done to avoid adverse risks associated with these solutions (select all that apply.)? Lung sounds Bowel sounds Blood pressure . Serum sodium level Serum potassium level

Lung sounds Blood pressure Serum sodium level Blood pressure, lung sounds, and serum sodium levels must be monitored frequently because of the risk for excess intravascular volume with hypertonic solutions.

What is the priority nursing objective of the therapeutic psychiatric environment for a confused client? Helping the client relate to others Making the hospital atmosphere more homelike Helping the client become accepted in a controlled setting Maintaining the highest level of safe, independent function

Maintaining the highest level of safe, independent function The therapeutic milieu is directed toward helping the client develop effective ways of functioning safely and independently. Helping the client relate to others is one small part of the overall objectives. The therapeutic milieu allows some items from home to make the client less anxious; however, the objective is not to duplicate a home situation. Helping the client become accepted in a controlled setting is a worthwhile objective but not as important as working toward the maximal degree of safe, independent function.

The nurse is caring for a patient who suddenly becomes lethargic and has an increase in the depth and rate of respirations has the following arterial blood gas (ABG) results: pH 7.32, PaO2 88 mm Hg, PaCO2 37 mm Hg, and HCO3 16 mEq/L. How should the nurse interpret these results? Metabolic acidosis Metabolic alkalosis Respiratory acidosis Respiratory alkalosis

Metabolic acidosis The pH and HCO3 indicate that the patient has a metabolic acidosis. The other options are incorrect.

You are admitting a patient with complaints of abdominal pain, nausea, and vomiting. A proximal bowel obstruction is suspected. Which acid-base imbalance do you anticipate in this patient? Metabolic acidosis Metabolic alkalosis Respiratory acidosis Respiratory alkalosis

Metabolic alkalosis Because gastric secretions are rich in HCl acid, the patient who is vomiting will lose a significant amount of gastric acid and be at an increased risk for metabolic alkalosis.

Which nursing intervention is most appropriate when caring for a patient with dehydration? Monitor skin turgor every shift. Auscultate lung sounds every 2 hours. Monitor daily weight and intake and output. Encourage the patient to reduce sodium intake.

Monitor daily weight and intake and output. Measuring weight is the most reliable means of detecting changes in fluid balance. Weight loss would indicate the dehydration is worsening, whereas weight gain would indicate restoration of fluid volume.

A patient with recurrent shortness of breath has just had a bronchoscopy. What is a priority nursing action immediately after the procedure? Monitor the patient for laryngeal edema. Assess the patient's level of consciousness. Monitor and manage the patient's level of pain. Assess the patient's heart rate and blood pressure.

Monitor the patient for laryngeal edema. Priorities for assessment are the patient's airway and breathing, both of which may be compromised after bronchoscopy by laryngeal edema. These assessment parameters supersede the importance of loss of consciousness (LOC), pain, heart rate, and blood pressure, although the nurse should also be assessing these.

The nurse is administering oral glucocorticoids to a patient with asthma. What assessment finding would the nurse identify as a therapeutic response to this medication? No observable respiratory difficulty or shortness of breath over the last 24 hours A decrease in the amount of nasal drainage and sneezing No sputum production, and a decrease in coughing episodes Relief of an acute asthmatic attack

No observable respiratory difficulty or shortness of breath over the last 24 hours Glucocorticoids (corticosteroids) decrease inflammation and prevent bronchospasm in the patient with asthma. The glucocorticoids are used to prevent problems. Anticholinergics decrease the allergic response and decrease sneezing and rhinorrhea. Antitussives are used to decrease cough, and mucolytics assist in the removal of mucus. Sympathomimetic agents (beta2 agonist) are used to relieve bronchospasm in an acute episode.

A community health nurse is preparing a course on protecting cognitive function. Which population group should the nurse target for teaching? Older male adults with diabetes Older female adults who are overweight Young adults living in school dormitories Adolescents attending summer camps

Older male adults with diabetes The primary risk factor for cognitive impairment is advancing age; males with a history of stroke or diabetes are at significant risk. Older females with a history of poor health, insomnia, and lack of social support are at risk for cognitive impairment, not those who are overweight. Risk factors for young adults include substance abuse and high-risk behaviors, not crowded living conditions. Adolescents who attend summer camp are not necessarily at risk for cognitive problems; adolescents who participate in high-risk behaviors would be at risk.

While assessing the cardiovascular status of a patient, the nurse performs auscultation. Which intervention should the nurse implement in the assessment during auscultation? Position the patient supine. Ask the patient to hold his or her breath. Palpate the radial pulse while auscultating the apical pulse. Use the bell of the stethoscope when auscultating S1 and S2.

Palpate the radial pulse while auscultating the apical pulse. To detect a pulse deficit, simultaneously palpate the radial pulse when auscultating the apical area. The diaphragm is more appropriate than the bell when auscultating S1 and S2. A sitting or side-lying position is most appropriate for cardiac auscultation. It is not necessary to ask the patient to hold his or her breath during cardiac auscultation.

You are caring for a patient admitted with a diagnosis of chronic obstructive pulmonary disease (COPD) who has the following arterial blood gas results: pH 7.33, PaO2 47 mm Hg, PaCO2 60 mm Hg, HCO3 32 mEq/L, and O2 saturation of 92%. What is the correct interpretation of these results? Fully compensated respiratory alkalosis Partially compensated respiratory acidosis Normal acid-base balance with hypoxemia Normal acid-base balance with hypercapnia

Partially compensated respiratory acidosis A low pH (normal, 7.35-7.45) indicates acidosis. In a patient with respiratory disease such as COPD, the patient retains carbon dioxide (normal, 35-45 mm Hg), which acts as an acid in the body. For this reason, the patient has respiratory acidosis. The elevated HCO3 indicates a partial compensation for the elevated CO2.

A client asks about the purpose of a pulse oximeter. The nurse explains that it is used to measure what? Respiratory rate Amount of oxygen in the blood Percentage of hemoglobin-carrying oxygen Amount of carbon dioxide in the blood

Percentage of hemoglobin-carrying oxygen The pulse oximeter measures the oxygen saturation of blood by determining the percentage of hemoglobin-carrying oxygen.

A patient is questioning the nurse about circulation and perfusion. What is the nurse's best response? Perfusion assists the body by preventing clots and increasing stamina. Perfusion assists the cell by delivering oxygen and removing waste products. Perfusion assists the heart by increasing the cardiac output. Perfusion assists the brain by increasing mental alertness.

Perfusion assists the cell by delivering oxygen and removing waste products. Perfusion delivers much needed oxygen to the cells of the body and then helps to remove waste products. Perfusion does not prevent clots, does not increase cardiac output, and does not increase mental alertness.

You are caring for a patient receiving calcium carbonate for the treatment of osteopenia. Which serum laboratory result would you identify as an adverse effect related to this therapy? Sodium falling to 138 mEq/L Potassium rising to 4.1 mEq/L Magnesium rising to 2.9 mg/dL Phosphorus falling to 2.1 mg/dL

Phosphorus falling to 2.1 mg/dL Calcium has an inverse relationship with phosphorus in the body. When phosphorus levels fall, calcium rises, and vice versa. Because hypercalcemia rarely occurs as a result of calcium intake, the patient's phosphorus falling to 2.1 mg/dL (normal, 2.4-4.4 mg/dL) may be a result of the phosphate-binding effect of calcium carbonate.

After assisting at the bedside with a thoracentesis, the nurse should continue to assess the patient for signs and symptoms of what? Bronchospasm Pneumothorax Pulmonary edema Respiratory acidosis

Pneumothorax Because thoracentesis involves the introduction of a catheter into the pleural space, there is a risk of pneumothorax. Thoracentesis does not carry a significant potential for causing bronchospasm, pulmonary edema, or respiratory acidosis.

The nurse is working with a patient who has been complaining of nausea and diarrhea. The nurse suspects dehydration. Which sign does the nurse expect to see? Flat neck veins when upright Decreased patellar reflexes Positive Trousseau sign Jugular vein distension

Positive Trousseau sign Trousseau sign is likely present in patients who have diarrhea or dehydration because dehydration can cause increased neuromuscular excitability. Flat neck veins when upright is incorrect because flat neck veins are not an expected finding in patient with diarrhea. Decreased patellar reflexes is incorrect because the reflexes would likely be increased or hyper in patients with diarrhea and dehydration. Jugular vein distension is incorrect because jugular vein distension is a sign of excess fluid volume.

A patient with a recent history of a dry cough has had a chest x-ray that revealed the presence of nodules. In an effort to determine whether the nodules are malignant or benign, what is the primary care provider likely to order? Thoracentesis Pulmonary angiogram CT scan of the patient's chest Positron emission tomography (PET)

Positron emission tomography (PET) PET is used to distinguish benign and malignant pulmonary nodules. Because malignant lung cells have an increased uptake of glucose, the PET scan (which uses an IV radioactive glucose preparation) can demonstrate increased uptake of glucose in malignant lung cells. This differentiation cannot be made using CT, a pulmonary angiogram, or thoracentesis.

The nurse knows the ventricular contractions are directly stimulated by which anatomic feature of the heart?

The Purkinje fibers move the electrical impulse or action potential through the walls of both ventricles triggering synchronized right and left ventricular contraction. The sinoatrial (SA) node initiates the electrical impulse that results in atrial contraction. The atrioventricular (AV) node receives the electrical impulse through internodal pathways. The bundle of His receives the impulse from the AV node.

The nurse is establishing a therapeutic environment for a patient admitted with dementia and influenza. Which intervention would be important for the nurse to implement? Keep a radio on all the time to provide sound for the patient. Decrease patient confusion by limiting verbal interactions. Limit family visits to one person for 30 minutes per day. Provide a quiet environment in a private room.

Provide a quiet environment in a private room. The patient experiencing dementia needs a quiet environment with a minimum of unfamiliar stimulation from a roommate. A patient with dementia does not need extra stimulation from having a radio on continually. The nurse should speak clearly and quietly to the patient before any procedure or assistance to decrease agitation. Family visits would be encouraged because family members are familiar to the patient and their presence increases a sense of security.

The patient has severe metabolic alkalosis. Which intervention has the highest priority? Raise the side rails on the patient's bed. Measure the urine output and skin turgor. Teach the family about metabolic alkalosis. Administer intravenous NaHCO3 as ordered.

Raise the side rails on the patient's bed. Severe metabolic alkalosis causes a decreased level of consciousness; raising the side rails is a safety intervention in that situation. Safety interventions are a higher priority than teaching. An order to administer intravenous NaHCO3 to a patient with metabolic alkalosis should be questioned because it would make the alkalosis worse. Urine output and skin turgor are part of the assessment for extracellular fluid volume (ECV) deficit, but this is not a high priority in this situation.

How is the relationship between the concepts of cognition and nutrition best expressed? Unidirectional Time dependent Indirect Reciprocal

Reciprocal The relationship between the concepts of cognition and nutrition is best expressed as reciprocal, meaning that cognition affects nutrition and nutrition affects cognition. An example of this reciprocity is the case of a person with impaired cognition who forgets to eat and drink. This leads to alterations in blood sugar and hydration status, which in turn further impair cognitive function.

While performing patient teaching regarding hypercalcemia, which statements are appropriate (select all that apply.)? Have patient restrict fluid intake to less than 2000 mL/day. Renal calculi may occur as a complication of hypercalcemia. Weight-bearing exercises can help keep calcium in the bones. The patient should increase daily fluid intake to 3000 to 4000 mL. Any heartburn can be managed with an as needed calcium-containing antacid.

Renal calculi may occur as a complication of hypercalcemia. Weight-bearing exercises can help keep calcium in the bones. The patient should increase daily fluid intake to 3000 to 4000 mL. Any heartburn can be managed with an as needed calcium-containing antacid.

A patient is admitted with metabolic acidosis. Which system is not functioning normally? Renal system Buffer system Endocrine system Respiratory system

Renal system When the patient has metabolic acidosis, the kidneys are not combining H+ with ammonia to form ammonium or eliminating acid with secretion of free hydrogen into the renal tubule. The buffer system neutralizes HCl acid by forming a weak acid. The hormone system is not directly related to acid-base balance. The respiratory system releases CO2 that combines with water to form hydrogen ions and bicarbonate. The hydrogen is then buffered by the hemoglobin.

The nurse in the emergency department is caring for a patient who is on a ventilator. The patient's blood gases report reveals a pH 7.48, PaO2 85 mm Hg, a PaCO2 32 mm Hg, and a HCO3 25 mEq/L. What is the nurse's interpretation of these results? Metabolic acidosis Metabolic alkalosis Respiratory acidosis Respiratory alkalosis

Respiratory alkalosis The pH indicates that the patient has alkalosis and the low PaCO2 indicates a respiratory cause. The other responses are incorrect based on the pH and the normal HCO3.

When assessing a patient admitted with nausea and vomiting, which finding best supports the nursing diagnosis of deficient fluid volume? Polyuria Bradycardia Restlessness Difficulty breathing

Restlessness Restlessness is an early cerebral sign that dehydration has progressed to the point where an intracellular fluid shift is occurring. If the dehydration is left untreated, cerebral signs could progress to confusion and later coma.

The patient's arterial blood gas results show the PaO2 at 65 mmHg and SaO2 at 80%. What early manifestations should the nurse expect to observe in this patient? Restlessness, tachypnea, tachycardia, and diaphoresis Unexplained confusion, dyspnea at rest, hypotension, and diaphoresis Combativeness, retractions with breathing, cyanosis, and decreased output Coma, accessory muscle use, cool and clammy skin, and unexplained fatigue

Restlessness, tachypnea, tachycardia, and diaphoresis With inadequate oxygenation, early manifestations include restlessness, tachypnea, tachycardia, and diaphoresis, decreased urinary output, and unexplained fatigue. The unexplained confusion, dyspnea at rest, hypotension, and diaphoresis; combativeness, retractions with breathing, cyanosis, and decreased urinary output; coma, accessory muscle use, cool and clammy skin, and unexplained fatigue occur as later manifestations of inadequate oxygenation.

A patient injured in an earthquake today when a wall fell on his legs received 9 units of blood an hour ago because he was hemorrhaging. Which laboratory value should the nurse check first when the report returns? Serum sodium Serum potassium Serum total calcium Serum magnesium

Serum potassium The patient has two major risk factors for hyperkalemia: massive sudden cell death from a crushing injury (potassium shift from cells into the extracellular fluid) and massive blood transfusion (rapid potassium intake). Although massive blood transfusion may cause calcium and magnesium ions to bind to citrate in the blood, thereby decreasing the physiologic availability of those ions, it does not decrease the total calcium or magnesium laboratory measurements. Clinically significant changes in serum sodium are the least likely in this patient.

The nurse is performing an assessment for a patient undergoing radiation treatment for breast cancer. What position should the nurse place the patient to best auscultate for signs of acute pericarditis? Supine without a pillow Sitting and leaning forward Left lateral side-lying position Head of bed at a 45-degree angle

Sitting and leaning forward A pericardial friction rub indicates pericarditis. To auscultate a pericardial friction rub, the patient should be sitting and leaning forward. The nurse will hear the pericardial friction rub at the end of expiration.

You receive a physician's order to change a patient's IV from D5½ NS with 40 mEq KCl/L to D5NS with 20 mEq KCl/L. Which serum laboratory values on this same patient best support the rationale for this IV order change? Sodium, 136 mEq/L; potassium, 3.6 mEq/L Sodium, 145 mEq/L; potassium, 4.8 mEq/L Sodium, 135 mEq/L; potassium, 4.5 mEq/L Sodium, 144 mEq/L; potassium, 3.7 mEq/L

Sodium, 135 mEq/L; potassium, 4.5 mEq/L The normal range for serum sodium is 135 to 145 mEq/L, and the normal range for potassium is 3.5 to 5.0 mEq/L. The change in the IV order decreases the amount of potassium and increases the amount of sodium. Therefore, for this order to be appropriate, the potassium level must be near the high end and the sodium level near the low end of their respective ranges.

A 50-yr-old woman with hypertension has a serum potassium level that has acutely risen to 6.2 mEq/L. Which type of order, if written by the health care provider, should the nurse question? Limit foods high in potassium Calcium gluconate IV piggyback Spironolactone (Aldactone) daily Administer intravenous insulin and glucose

Spironolactone (Aldactone) daily Spironolactone (Aldactone) is a potassium-sparing diuretic that inhibits the exchange of sodium for potassium in the distal renal tubule and helps to prevent potassium loss. Spironolactone is contraindicated in a patient with hyperkalemia (serum potassium >5.0 mEq/L). Management of patients with hyperkalemia may include limiting foods high in potassium, administering IV insulin and glucose, administering IV calcium gluconate, changing to potassium-wasting diuretics (e.g., furosemide [Lasix]), hemodialysis, administering sodium polystyrene sulfonate (Kayexalate), and IV fluid administration.

A patient is having the arterial blood gas (ABG) measured. What would the nurse identify as the parameters to be evaluated by this test? Ratio of hemoglobin and hematocrit Status of acid-base balance in arterial blood Adequacy of oxygen transport Presence of a pulmonary embolus

Status of acid-base balance in arterial blood The ABG results will indicate the acid-base balance of the arterial blood and the partial pressure of oxygen and carbon dioxide. The ABG does not reveal the ratio of hemoglobin and hematocrit, the adequacy of oxygen transport to the cells, or the presence of a pulmonary embolus.

The nurse is reviewing the needs of a patient with cognitive impairment. What is the priority concern that the nurse should address for this patient? Promoting at least 6 hours of sleep a night Encouraging an oral intake of 1200 calories per day Managing the patient's pain from arthritis Supervising medication administration

Supervising medication administration Safety is the priority concern for the cognitively impaired patient; safely taking medication addresses safety needs for the patient. Sleep, nutrition, and management of pain are important components of the patient's care and can affect overall health, but safety is the highest priority.

A patient with aortic valve stenosis is being admitted for valve replacement surgery. Which assessment finding documented by the nurse is indicative of this condition? Pulse deficit Systolic murmur Distended neck veins Splinter hemorrhages

Systolic murmur The turbulent blood flow across a diseased valve results in a murmur. Aortic stenosis produces a systolic murmur. A pulse deficit indicates a cardiac dysrhythmia, most commonly atrial fibrillation. Distended neck veins may be caused by right-sided heart failure. Splinter hemorrhages occur in patients with infective endocarditis.

The nurse is caring for an older adult patient. What age-related cardiovascular changes should the nurse assess for when providing care for this patient (select all that apply.)? Systolic murmur Diminished pedal pulses Increased maximal heart rate Decreased maximal heart rate Increased recovery time from activity

Systolic murmur Diminished pedal pulses Decreased maximal heart rate Increased recovery time from activity Well-documented cardiovascular effects of the aging process include valvular rigidity leading to systolic murmur, arterial stiffening leading to diminished pedal pulses or possible increased blood pressure, and an increased amount of time that is required for recovery from activity. Maximal heart rate tends to decrease rather than increase with age related to cellular aging and fibrosis of the conduction system.

What important step should the community nurse take when dealing with older adults with a confusional states problem? Select all that apply. The nurse should provide a protective environment. The nurse should monitor blood pressure and weight. The nurse should recommend applicable community resources. The nurse should demonstrate proper hygiene to the primary caretaker. The nurse should educate about polypharmacy and drug-drug and drug-food interactions

The nurse should provide a protective environment. The nurse should recommend applicable community resources. The nurse should demonstrate proper hygiene to the primary caretaker. When dealing with older adults with a confusional states problem, the nurse should ideally provide a protective environment for the client. In addition, the nurse should recommend applicable community resources like adult day care, home care aides, and homemaker services. When dealing with community-dwelling older adults with a confusional states problem, the nurse should assist with adequate personal hygiene, nutrition, and hydration. When dealing with the community-dwelling older adults with a hypertension problem, the nurse should monitor blood pressure and weight. When dealing with the community-dwelling older adults with a medication use and abuse problem, the nurse should educate about polypharmacy and drug-drug and drug-food interactions.

The nurse observes that during morning care the patient is complaining of leg pain when ambulating to the bathroom. The nurse assists the patient back into bed and notices that the patient's leg pain is relieved. Further assessment reveals bilateral pedal edema. The nurse knows that the cause of the patient's leg pain is most likely which of the following: The pain indicates an inadequate amount of blood to transport oxygen to meet the demands of leg muscles. The pain indicates a muscle spasm. The patient is having a myocardial infarction. The pain is due to over-exertion during morning care.

The pain indicates an inadequate amount of blood to transport oxygen to meet the demands of leg muscles. Impaired perfusion often results in leg pain as related to peripheral arterial disease (PAD). PAD leg pain is often relieved with rest and worsens with walking. Leg pain that is relieved with rest is called intermittent claudication and means that there is an inadequate supply of blood being transported to the muscles. Edema also develops from the obstruction of venous blood flow.Although pain is common during a muscle spasm, it is usually not relieved with rest. During a myocardial infarction, pain is often felt in the chest and not in the lower extremities.Although pain may occur from exercise, acute leg pain with the presence of edema indicates a perfusion problem and warrants further investigation.

A 72-year-old man presents to the emergency room. The patient appears diaphoretic and anxious, and has noted peripheral edema. The patient's vital signs are blood pressure of 100/40, heart rate of 130 and irregular, and respiratory rate of 26. How does the nurse interpret these findings? The patient is having a myocardial infarction. The patient has impaired central perfusion. The patient has a virus. Pain medication should be administered to this patient.

The patient has impaired central perfusion. This patient has the classic symptoms of impaired central perfusion. Central perfusion occurs when cardiac output is optimal and blood is pumped to all of the organs and tissues from the arteries, through the capillaries, and then back to the heart through the veins. The nurse needs to administer oxygen. Chest pain is often present with myocardial infarction, along with elevated blood pressure readings and electrocardiogram changes. Viral illness commonly presents with other symptoms such as body ache or gastrointestinal issues, and typically has little or no effect on the heart rate. Pain management is not indicated for patients who do not present with pain. Also, the question is asking what assessment the nurse has made, and is not asking about interventions.

The nurse is caring for a patient with emphysema. The patient is complaining of shortness of breath and dyspnea on minimal exertion. Which assessment finding alerts the nurse that the patient is going into respiratory failure? The patient has bibasilar lung crackles. The patient is sitting in the tripod position. The patient's respirations have decreased from 30 to 10 breaths/minute. The patient's pulse oximetry indicates an O2 saturation of 91%.

The patient's respirations have decreased from 30 to 10 breaths/minute. A decrease in respiratory rate in a patient with respiratory distress suggests the onset of fatigue and a high risk for respiratory arrest. Therefore immediate action such as positive pressure ventilation is needed. Patients who are experiencing respiratory distress frequently sit in the tripod position because it decreases the work of breathing. Crackles in the lung bases may be the baseline for a patient with chronic obstructive pulmonary disease (COPD). An oxygen saturation of 91% is common in patients with COPD and will provide adequate gas exchange and tissue oxygenation.

You are caring for a patient admitted with diabetes mellitus, malnutrition, and a massive GI bleed. In analyzing the morning lab results, the nurse understands that a potassium level of 5.5 mEq/L could be caused by which factors in this patient (select all that apply.)? The potassium level may be increased if the patient has nephropathy. The patient has been eating excessive amounts of foods that increase potassium levels. The patient may be excreting extra sodium and retaining potassium secondary to malnutrition. There may be excess potassium being released into the blood as a result of massive blood transfusion. The potassium level may be increased because of dehydration that accompanies high blood glucose levels.

The potassium level may be increased if the patient has nephropathy. There may be excess potassium being released into the blood as a result of massive blood transfusion. The potassium level may be increased because of dehydration that accompanies high blood glucose levels. Hyperkalemia may result from hyperglycemia, renal insufficiency, or cell death. Diabetes mellitus, along with the stress of hospitalization and illness, can lead to hyperglycemia. Renal insufficiency is a complication of diabetes. Because malnutrition does not cause sodium excretion accompanied by potassium retention, it is not a contributing factor to this patient's potassium level. Stored hemolyzed blood can cause hyperkalemia when large amounts are transfused rapidly. The patient with a massive GI bleed would have an nasogastric tube and not be eating.

The nurse informs the patient that she must wear intermittent sequential compression stockings after a surgical procedure. What is an appropriate rationale for nurse to give to the patient for the use of the device? The socks keep the legs warm while the patient is not moving much. The socks maintain the blood flow to the legs while the patient is on bed rest. The socks keep the blood pressure down while the patient is stressed after surgery. The socks provide compression of the veins to keep the blood moving back to the heart.

The socks provide compression of the veins to keep the blood moving back to the heart. Intermittent sequential compression stockings provide compression of the veins while the patient is not using skeletal muscles to compress the veins, which keeps the blood moving back to the heart and prevents blood pooling in the legs that could cause deep vein thrombosis. The warmth is not important. Blood flow to the legs is not maintained. Blood pressure is not decreased with the use of intermittent sequential compression stockings.

While auscultating the patient's heart sounds with the bell of the stethoscope, the nurse hears these sounds. How should the nurse document what is heard? Diastolic murmur Third heart sound (S3) Fourth heart sound (S4) Normal heart sounds (S1, S2)

Third heart sound (S3) The third heart sound is heard closely after the S2 and is known as a ventricular gallop because it is a vibration of the ventricular walls associated with decreased compliance of the ventricles during filling. It occurs with left ventricular failure. Murmurs sound like turbulence between normal heart sounds and are caused by abnormal blood flow through diseased valves. The S4 heart sound is a vibration caused by atrial contraction, precedes the S1, and is known as an atrial gallop. The normal S1 and S2 are heard when the valves close normally.

In assessment of the patient with acute respiratory distress, what should the nurse expect to observe (select all that apply.)? Cyanosis Tripod position Kussmaul respirations Accessory muscle use Increased AP diameter

Tripod position Accessory muscle use Tripod position and accessory muscle use indicate moderate to severe respiratory distress. Cyanosis may be related to anemia, decreased oxygen transfer in the lungs, or decreased cardiac output. Therefore, it is a nonspecific and unreliable indicator of only respiratory distress. Kussmaul respirations occur when the patient is in metabolic acidosis to increase CO2 excretion. Increased AP diameter occurs with lung hyperinflation from chronic obstructive pulmonary disease, cystic fibrosis, or with advanced age.

A patient presents to the emergency department with reports of chest pain for 3 hours. What component of his blood work is most clearly indicative of a myocardial infarction (MI)? CK-MB Troponin Myoglobin C-reactive protein

Troponin Troponin is the biomarker of choice in the diagnosis of MI, with sensitivity and specificity that exceed those of CK-MB and myoglobin. CRP levels are not used to diagnose acute MI.

The nurse is performing an assessment for a patient with fatigue and shortness of breath. Auscultation of the heart reveals the presence of a murmur. What is this assessment finding indicative of? Increased viscosity of the patient's blood Turbulent blood flow across a heart valve Friction between the heart and the myocardium A deficit in heart conductivity that impairs normal contractility

Turbulent blood flow across a heart valve Turbulent blood flow across the affected valve results in a murmur. A murmur is not a direct result of variances in blood viscosity, conductivity, or friction between the heart and myocardium.

Ideomotor apraxia is classified as a deficit in which cognitive area? Memory Language Thought process Visuospatial

Visuospatial Ideomotor apraxia is an abnormality affecting the visuospatial cognitive area. Apraxia is the inability to perform purposeful movements or manipulate objects despite intact sensory and motor abilities. Ideomotor apraxia is a specific type of apraxia in which there is an inability to translate an idea into action.

A patient is admitted to the emergency department with dehydration. Arterial blood gas (ABG) results reveal that the patient has metabolic acidosis. Which of the following signs or symptoms is the most likely cause of this imbalance? Hypoventilation Vomiting and diarrhea Serum potassium is 5.1 mEq/L. Arterial oxygen saturation is 91%.

Vomiting and diarrhea Vomiting and diarrhea cause too much acid, the buffers have been overwhelmed, and body fluids have too much acid. Acid excretion is not able to keep up with acid production or intake. Hypoventilation leads to respiratory acidosis Hyperventilation leads to respiratory alkalosis. Oxygen saturations below 95% indicate that the patient may need supplemental oxygen and may contribute to imbalance; however, a low reading alone is not enough to determine the cause.

While caring for a patient with metastatic bone cancer, which clinical manifestations would alert the nurse to the possibility of hypercalcemia in this patient (select all that apply.)? Weakness Paresthesia Facial spasms Muscle tremors Depressed reflexes

Weakness Depressed reflexes Signs of hypercalcemia are lethargy, fatigue, weakness, depressed reflexes, muscle flaccidity, heart block, anorexia, nausea, and vomiting. Paresthesia, facial spasms, and muscle tremors are symptoms of hypocalcemia.

The nurse should ask which of the following questions to detect the risk factors for metabolic acidosis? (Select all that apply.) Have you been vomiting today? When did your kidneys stop working? How long have you had diarrhea? Are you still feeling short of breath? What type of antacid did you take? Which weight loss diet are you using?

When did your kidneys stop working? How long have you had diarrhea? Which weight loss diet are you using? Risk factors for metabolic acidosis include decreased excretion of metabolic acid from oliguria or anuria (kidneys are not working); excessive production of metabolic acid from starvation ketoacidosis (inappropriate weight loss diet); and loss of bicarbonate from diarrhea. Vomiting (loss of acid) causes metabolic alkalosis, as does overusing bicarbonate antacids. Shortness of breath might be related to a cause of respiratory acidosis.

A nurse is preparing to teach a group of women in a community volunteer group about heart disease. What should the nurse include in the teaching plan? Women are less likely to delay seeking treatment than men. Women are more likely to have noncardiac symptoms of heart disease. Women are often less ill when presenting for treatment of heart disease. Women experience more symptoms of heart disease at a younger age than men.

Women are more likely to have noncardiac symptoms of heart disease. Women often have atypical angina symptoms and nonpain symptoms. Women experience the onset of heart disease about 10 years later than men. Women are often more ill on presentation and delay longer in seeking care than men.

The nursing care for a patient with hyponatremia and fluid volume excess includes a. fluid restriction. b. administration of hypotonic IV fluids. c. administration of a cation-exchange resin. d. placement of an indwelling urinary catheter.

a. fluid restriction. Rationale: In hyponatremia that is caused by water excess, fluid restriction often is all that is needed to treat the problem. The patient would only require an indwelling urinary catheter if the patient is unable to assist with maintaining an accurate output record.

The lungs act as an acid-base buffer by a. increasing respiratory rate and depth when CO2 levels in the blood are high, reducing acid load. b. increasing respiratory rate and depth when CO2 levels in the blood are low, reducing base load. c. decreasing respiratory rate and depth when CO2 levels in the blood are high, reducing acid load. d. decreasing respiratory rate and depth when CO2 levels in the blood are low, increasing acid load.

a. increasing respiratory rate and depth when CO2 levels in the blood are high, reducing acid load. Rationale: As a compensatory mechanism, the respiratory system acts on the CO2 + H2O side of the reaction by altering the rate and depth of breathing to "blow off" (through hyperventilation) or "retain" (through hypoventilation) CO2.

During administration of a hypertonic IV solution, the mechanism involved in equalizing the fluid concentration between ECF and the cells is a. osmosis. b. diffusion. c. active transport. d. facilitated diffusion.

a. osmosis. Rationale: Osmosis is the movement of water between two compartments separated by a semipermeable membrane. Water moves through the membrane from an area of low solute concentration to an area of high solute concentration.

. An older woman was admitted to the medical unit with GI bleeding and fluid volume deficit. Clinical manifestations of this problem are (select all that apply) a. weight loss. b. dry oral mucosa. c. full bounding pulse. d. engorged neck veins. e. decreased central venous pressure.

a. weight loss. b. dry oral mucosa. e. decreased central venous pressure. Rationale: Body weight loss, especially sudden change, is an excellent indicator of overall fluid volume loss. Other clinical manifestations of fluid volume deficit include dry mucous membranes and a decreased central venous pressure, which reflect fluid volume loss.

The nurse is caring for a patient who had abdominal surgery yesterday. Today the patient's lung sounds in the lower lobes are diminished. The nurse knows this could be related to the occurrence of pain atelectasis pneumonia pleural effusion.

atelectasis Postoperatively, there is an increased risk for atelectasis from anesthesia as well as restricted breathing from pain. Without deep breathing to stretch the alveoli, surfactant secretion to hold the alveoli open is not promoted. Pneumonia will occur later after surgery. Pleural effusion occurs because of blockage of lymphatic drainage or an imbalance between intravascular and oncotic fluid pressures, which is not expected in this case.

A patient is admitted with severe dyspnea, a history of heart failure, and chronic obstructive lung disease. Which diagnostic study would the nurse expect to be elevated if the cause of dyspnea was cardiac related? Serum potassium Serum homocysteine High-density lipoprotein b-type natriuretic peptide (BNP)

b-type natriuretic peptide (BNP) Elevation of BNP indicates the presence of heart failure. Elevations help to distinguish cardiac versus respiratory causes of dyspnea. Elevated potassium, homocysteine, or HDL levels may indicate increased risk for cardiovascular disorders but do not indicate that cardiac disease is present.

The nurse expects the long-term treatment of a patient with hyperphosphatemia secondary to renal failure will include a. fluid restriction. b. calcium supplements. c. magnesium supplements. d. increased intake of dairy products.

b. calcium supplements. Rationale: The major conditions that can lead to hyperphosphatemia are acute kidney injury and chronic kidney disease that alter the ability of the kidneys to excrete phosphate. For the patient with renal failure, long-term measures to reduce serum phosphate levels include phosphate-binding agents or gels, such as calcium carbonate, fluid replacement therapy, and dietary phosphate restrictions.

The nurse is unable to flush a central venous access device and suspects occlusion. The best nursing intervention would be to a. apply warm moist compresses to the insertion site. b. attempt to force 10 mL of normal saline into the device. c. place the patient on the left side with head-down position. d. instruct the patient to change positions, raise arm, and cough.

d. instruct the patient to change positions, raise arm, and cough. Rationale: Interventions for catheter occlusion include instructing the patient to change position, raise an arm, and cough; assessing for and alleviating clamping or kinking of the tube; flushing the catheter with normal saline through a 10-mL syringe (do not force flush); using fluoroscopy to determine cause and site of occlusion; and instilling anticoagulant or thrombolytic agents.

A patient has the following arterial blood gas results: pH 7.52, PaCO2 30 mm Hg, HCO3− 24 mEq/L. The nurse determines that these results indicate a. metabolic acidosis. b. metabolic alkalosis. c. respiratory acidosis. d. respiratory alkalosis.

d. respiratory alkalosis. Rationale: Respiratory alkalosis (carbonic acid deficit) occurs with hyperventilation. The primary cause of respiratory alkalosis is hypoxemia from acute pulmonary disorders. Anxiety, central nervous system (CNS) disorders, and mechanical overventilation also increase ventilation rate and decrease the partial pressure of arterial carbon dioxide (PaCO2). This leads to a decrease in carbonic acid level and to alkalosis.

During the postoperative care of a 76-year-old patient, the nurse monitors the patient's intake and output carefully, knowing that the patient is at risk for fluid and electrolyte imbalances primarily because a. older adults have an impaired thirst mechanism and need reminding to drink fluids. b. water accounts for a greater percentage of body weight in the older adult than in younger adults. c. older adults are more likely than younger adults to lose extracellular fluid during surgical procedures. d. small losses of fluid are significant because body fluids account for 45% to 50% of body weight in older adults.

d. small losses of fluid are significant because body fluids account for 45% to 50% of body weight in older adults. Rationale: In the older adult, body water content averages 45% to 50% of body weight; therefore small losses can lead to a greater risk of fluid-related problems.

The nurse is caring for a 76-yr-old woman admitted to the medical unit with hypernatremia and dehydration after prolonged fever. The best beverage to offer the patient is malted milk. orange juice. tomato juice. hot chocolate.

orange juice. Orange juice would be the safest option because it has the least amount of sodium (~2 mg in 8 oz). Hot chocolate has approximately 75 mg sodium in 8 ounces. Tomato juice has approximately 650 mg sodium in 8 oz. Malted milk has approximately 625 mg sodium in 8 oz.

A patient is having her first severe, acute asthma episode. It began 2 hours ago. What blood gas values should the nurse expect to see? pH high, PaCO2 high, HCO3- high pH low, PaCO2 low, HCO3- low pH low, PaCO2 high, HCO3- high pH low, PaCO2 high, HCO3- normal

pH low, PaCO2 high, HCO3- normal A severe acute asthma episode impairs the excretion of carbonic acid, causing respiratory acidosis with a high PaCO2 and a low pH. Renal compensation takes longer than 2 hours to occur, so the respiratory acidosis is uncompensated, leaving the HCO3- normal. A high pH occurs with alkalosis, not acidosis. ANSs that include abnormal levels of HCO3- are not correct for the 2-hour time frame.

You are caring for a patient admitted with an exacerbation of asthma. After several treatments, the ABG results are pH 7.40, PaCO2 40 mm Hg, HCO3 24 mEq/L, PaO2 92 mm Hg, and O2 saturation of 99%. You interpret these results as metabolic acidosis. respiratory acidosis. respiratory alkalosis. within normal limits.

within normal limits. The normal pH is 7.35 to 7.45. Normal PaCO2 levels are 35 to 45 mm Hg, and HCO3 is 22 to 26 mEq/L. Normal PaO2 is >80 mm Hg. Normal oxygen saturation is >95%. Because the patient's results all fall within these normal ranges, the nurse can conclude that the patient's blood gas results are within normal limits.

Which statement by the wife of a patient with Alzheimer's disease (AD) demonstrates an accurate understanding of her husband's medication regimen? • "I'm really hoping his medications will slow down his mental losses." • "We're both holding out hope that this medication will cure his disease." • "I know that this won't cure him, but we learned that it might prevent a bodily decline while he declines mentally." • "I learned that if we are vigilant about his medication schedule, he may not experience the physical effects of his disease."

• "I'm really hoping his medications will slow down his mental losses." There is presently no cure for AD, and drug therapy aims at improving or controlling decline in cognition. Medications do not directly address the physical manifestations of AD.

A patient is experiencing periods of confusion, and the family is concerned. The patient's son asks the nurse for an explanation and recommendation. What is the nurse's best response? • "Your father may be having mini-strokes; I will notify his physician." • "Your father is just confused about some things since he is in the hospital." • "The confusion will pass. Your father just has to get up and move around." • "Talk with your father about past events, and that will help with the confusion."

• "Your father may be having mini-strokes; I will notify his physician." Periods of confusion may be related to mini-strokes, or transient ischemic attacks (TIAs). Confusion during hospitalization does not occur with every patient. Talking with the patient or thinking the confusion may pass is not a viable solution. The patient should be assessed and the reason for the confusion identified.

The home care nurse is visiting patients in the community. Which patient is exhibiting an early warning sign of Alzheimer's disease (AD)? • A 65-yr-old male patient does not recognize his family members and close friends • A 59-yr-old female patient misplaces her purse and jokes about having memory loss • A 79-yr-old male patient is incontinent and not able to perform hygiene independently • A 72-yr-old female patient is unable to locate the address where she has lived for 10 years

• A 72-yr-old female patient is unable to locate the address where she has lived for 10 years An early warning sign of AD is disorientation to time and place such as geographic disorientation. Occasionally misplacing items and joking about memory loss are examples of normal forgetfulness. Impaired ability to recognize family and close friends is a clinical manifestation of middle or moderate dementia (or AD). Incontinence and inability to perform self-care activities are clinical manifestations of severe or late dementia (or AD).

The nurse would anticipate that which of the following patients will need to be treated with insertion of a chest tube? • A patient with asthma and severe shortness of breath • A patient undergoing a bronchoscopy for a biopsy • A patient with a pleural effusion requiring fluid removal • A patient experiencing a problem with a pneumothorax

• A patient experiencing a problem with a pneumothorax When air is allowed to enter the pleural space, the lung will collapse and a chest tube will be inserted to remove the air and reestablish negative pressure in the pleural space. Patients with asthma do not require a chest tube. A bronchoscopy is done to evaluate the bronchi and lungs and to obtain a biopsy. A thoracentesis may be done to remove fluid from the pleural space. A chest tube may be inserted if there are complications from the thoracentesis or for the bronchoscopy.

Which patient should receive a depression assessment first? • A patient in the early stages of Alzheimer's disease • A patient who is in the final stages of Alzheimer's disease • A patient experiencing delirium secondary to dehydration • A patient who has become delirious following an atypical drug response

• A patient in the early stages of Alzheimer's disease Patients in the early stages of Alzheimer's disease are particularly susceptible to depression because they are acutely aware of their cognitive changes and the expected disease trajectory. Delirium is typically a shorter term health problem that does not typically pose a heightened risk of depression.

When caring for a patient with Alzheimer's disease, which task could be delegated to the LPN/LVN on the team? • Administer enteral feedings via a gastrostomy tube. • Teach patient and caregivers memory enhancement aids. • Use bed alarms and frequent monitoring to decrease fall risk. • Make referrals for community services such as adult day care.

• Administer enteral feedings via a gastrostomy tube. Administering enteral feedings via a gastrostomy tube is within the scope of practice for the LPN/LVN. The RN will be responsible for individualized teaching and patient referrals. The UAP will be able to use bed alarms and frequently monitor the patient.

A 59-yr-old female patient with a frontotemporal lobar dementia has difficulty with verbal expression. While her husband was at work, she walked to the gas station for a soda but did not understand the request for payment. What can the nurse suggest to keep the patient safe? • Assisted living • Adult day care • Advance directives • Monitor for behavioral changes

• Adult day care To keep the patient safe during the day while the husband is at work, an adult day care facility would be the best choice. This patient would not need assisted living. Advance directives are important but are not related to her safety. Monitoring for behavioral changes will not keep her safe during the day.

Benzodiazepines are indicated in the treatment of delirium caused by which condition? • Polypharmacy • Cerebral hypoxia • Alcohol withdrawal • Electrolyte imbalances

• Alcohol withdrawal Benzodiazepines can be used to treat delirium associated with sedative and alcohol withdrawal. However, these drugs may worsen delirium caused by other factors and must be used cautiously. Polypharmacy, cerebral hypoxia, and electrolyte imbalances are not treated with benzodiazepines.

Which patient has the greatest risk of developing delirium? • A patient with fibromyalgia whose chronic pain has recently worsened • A patient with a fracture who has spent the night in the emergency department • An older patient whose recent computed tomography shows brain atrophy • An older patient who takes multiple medications to treat various health problems

• An older patient who takes multiple medications to treat various health problems Polypharmacy is implicated in many cases of delirium, and this phenomenon is especially common among older adults. Brain atrophy, if associated with cognitive changes, is indicative of dementia. Alterations in sleep and environment, as well as pain, may cause delirium, but this is less of a risk than in an older adult who takes multiple medications.

The patient is having some increased memory and language problems. What diagnostic tests will be done before this patient is diagnosed with Alzheimer's disease (select all that apply.)? • Urinalysis • Chest x-ray • MRI of the head • Liver function tests • Neuropsychologic testing • Blood urea nitrogen and serum creatinine

• Blood urea nitrogen and serum creatinine • Neuropsychologic testing • MRI of the head • Liver function tests Because there is no definitive diagnostic test for Alzheimer's disease, and many conditions can cause manifestations of dementia, testing must be done to eliminate any other causes of cognitive impairment. These include urinalysis to eliminate a urinary tract infection, an MRI to eliminate brain tumors, liver function tests to eliminate encephalopathy, BUN and serum creatinine to rule out renal dysfunction, and neuro-psychologic testing to assess cognitive function. A chest x-ray examination is not used to investigate an alternate cause of memory or language problems.

Which nursing intervention is most appropriate when caring for patients with dementia? • Avoid direct eye contact. • Lovingly call the patient "honey" or "sweetie." • Give simple directions, focusing on one thing at a time. • Treat the patient according to his or her age-related behavior.

• Give simple directions, focusing on one thing at a time. When dealing with patients with dementia, tasks should be simplified, giving directions using gestures or pictures and focusing on one thing at a time. It is best to treat these patients as adults, with respect and dignity, even when their behavior is childlike. The nurse should use gentle touch and direct eye contact. Calling the patient "honey" or "sweetie" can be condescending and does not demonstrate respect.

Exercise and activity are included in a cardiac rehabilitation program for which purposes? • Increase cardiac output • Increase serum lipids • Increase blood pressure • Increase blood flow to the arteries • Increase muscle mass • Increase flexibility

• Increase blood flow to the arteries • Increase muscle mass • Increase flexibility • Increase cardiac output A cardiac rehabilitation program seeks to increase cardiac output, blood flow to the arteries, muscle mass, and flexibility. The rehabilitation program does not want to increase serum lipids or blood pressure.

The nurse has administered a dose of risperidone (Risperdal) to a patient with delirium. What is the intended effect of the medication? • Lying quietly in bed • Alleviation of depression • Reduction in blood pressure • Disappearance of confusion

• Lying quietly in bed Risperidone is an antipsychotic drug that reduces agitation and produces a restful state in patients with delirium. However, it should be used with caution. Antidepressant medications treat depression, and antihypertensive medications treat hypertension. However, there are no medications that will cause confusion to disappear in a patient with delirium.

Unlicensed assistive personnel (UAP) working for a home care agency report a change in the alertness and language of an 82-yr-old female patient. The home care nurse plans a visit to evaluate the patient's cognitive function. Which assessment would be most appropriate? • Glasgow Coma Scale (GCS) • Confusion Assessment Method (CAM) • Mini-Mental State Examination (MMSE) • National Institutes of Health Stroke Scale (NIHSS)

• Mini-Mental State Examination (MMSE) The MMSE is a commonly used tool to assess cognitive function. Cognitive testing is focused on evaluating memory, ability to calculate, language, visual-spatial skills, and degree of alertness. The CAM is used to assess for delirium. The GCS is used to assess the degree of impaired consciousness. The NIHSS is a neurologic examination stroke scale used to evaluate the effect of acute cerebral infarction on the levels of consciousness, language, neglect, visual field loss, extraocular movement, motor strength, ataxia, dysarthria, and sensory loss.

The nurse is assessing a female patient at the neighborhood clinic. The patient is complaining of "feeling tired all the time." The nurse knows that fatigue may be an underlying symptom of which condition? • Ischemia • Pneumonia • Myocardial infarction • Peptic ulcer disease

• Myocardial infarction Fatigue is an atypical symptom of myocardial infarction in women. Ischemia is associated with pain. Pneumonia is associated with pain and shortness of breath. Peptic ulcer disease is associated with pain and intestinal discomfort.

When providing community health care teaching regarding the early warning signs of Alzheimer's disease (AD), which signs should the nurse ask family members to report (select all that apply.)? • Misplacing car keys • Losing sense of time • Difficulty performing familiar tasks • Problems with performing basic calculations • Momentarily forgets an acquaintance's name • Becoming lost in a usually familiar environment

• Problems with performing basic calculations • Difficulty performing familiar tasks • Losing sense of time • Becoming lost in a usually familiar environment Difficulty performing familiar tasks, problems with performing basic calculations, losing sense of time, and becoming lost in a usually familiar environment are all part of the early warning signs of AD. Misplacing car keys and momentarily forgetting a name is a normal frustrating event for many people.

The nurse in the long-term care facility cares for a 70-yr-old man with late-stage dementia who is undernourished and has problems chewing and swallowing. What should the nurse include in the plan of care for this patient? • Turn on the television to provide a distraction during meals. • Provide thickened fluids and moist foods in bite-size pieces. • Limit fluid intake during scheduled meals to prevent aspiration.. • Allow the patient to select favorite foods from the menu choices.

• Provide thickened fluids and moist foods in bite-size pieces. If patients with dementia have problems chewing or swallowing, pureed foods, thickened liquids, and nutritional supplements should be provided. Foods that are easy to swallow are moist and should be in bite-size pieces. Distractions at mealtimes, including the television, should be avoided. Fluids should not be limited but offered frequently; fluids should be thickened. Patients with late-stage dementia have difficulty understanding words and would not have the cognitive ability to select menu choices.

A 78-yr-old woman was transferred to the intensive care unit after emergency abdominal surgery. The nurse notes the patient is disoriented and confused, has incoherent speech, is restless, and agitated. Which action by the nurse is most appropriate? • Reorient the patient. • Notify the physician. • Document the findings. • Administer lorazepam (Ativan).

• Reorient the patient. The patient is exhibiting clinical manifestations of delirium. Care of the patient with delirium is focused on eliminating precipitating factors and protecting the patient from harm. Give priority to creating a calm and safe environment. The nurse should stay at the bedside and provide reassurance and reorienting information as to place, time, and procedures. The nurse should reduce environmental stimuli, including noise and light levels. Avoid the use of chemical and physical restraints if possible.

What is the most significant modifiable risk factor for the development of impaired gas exchange? • Age • Tobacco use • Drug overdose • Prolonged immobility

• Tobacco use Tobacco use is the most preventable cause of death and disease and is the most important risk factor in the development of impaired gas exchange. Age is not a modifiable risk factor. Drug overdose and immobility both contribute to impaired gas exchange but are not as significant as tobacco use.

A patient is diagnosed with the mild cognitive impairment stage of Alzheimer's disease. What nursing intervention should the nurse use with the patient? • Communicate using a letter or picture board. • Treat disruptive behavior with antipsychotic drugs. • Use a calendar and family pictures as memory aids. • Apply a wander guard mechanism to keep the patient in the area.

• Use a calendar and family pictures as memory aids. The patient with mild cognitive impairment will have problems with memory, language, or another essential cognitive function that is severe enough to be noticeable to others but does not interfere with activities of daily living. A calendar and family pictures for memory aids will help this patient. This patient should not yet have disruptive behavior or get lost easily. Using a writing board will not help this patient with communication.

The nurse is assessing a patient for sleep patterns. The patient reports that he has trouble sleeping when lying flat. The best response from the nurse is • open a window to let fresh air into the room. • use nasal strips to assist with breathing. • sleep in a side-lying position. • use pillows to prop yourself up while sleeping.

• use pillows to prop yourself up while sleeping. Using pillows to prop himself up during sleep allows the patient to breathe more easily and comfortably. Nasal strips will help with breathing, but they do not always bring relief when one is lying flat. Sleeping in a side position or opening a window does not help one to breathe more easily when one is lying flat.


Kaugnay na mga set ng pag-aaral

Tennessee Health Insurance License Review ExamFX

View Set

Chapter 17- Gastrointestinal and Urologic Emergency

View Set

Honan-Chapter 41: Nursing Management: Patients With Musculoskeletal Disorders

View Set

MGT-2013 Summer Course Test 1 Chapter's 1-8

View Set